CET108Revised ANSHULGARG

You might also like

Download as pdf or txt
Download as pdf or txt
You are on page 1of 93

Login ID:8FA03376/Student Name:ANSHULGARG/Overall Score:7

Student ScoreCard
Score:0

Percentile: NA

Test of Logical Reasoning

Score:5

Percentile: NA

Test of Quantitative Aptitude

Score:0

Percentile: NA

Test of VA & RC

Score:2

Percentile: NA

Overall: Overall

Score:7

Percentile: NA

ite
d

Test of Abstract Reasoning

2)

[2]

3)

[3]

4)

[4]

ou

[1]

es

1)

rc

es

Pr

iv
at
e

Li
m

Question: 1
The second figure in the first unit of the problem figures bears a certain relationship to the first figure. Similarly, one of the figures in the
second unit bears the same relationship to the figure represented by the question mark (?). You are therefore to locate the figure which
would replace the question mark (?) from the options [1], [2], [3] & [4].

ng

Explanation:
Analyse the relationship between the figures in three parts - the tail, the arrow head and the middle part i.e., line correcting the tail and
arrow head. Hence, [2].

IM

Le

ar

ni

Question: 2
The second figure in the first unit of the problem figures bears a certain relationship to the first figure. Similarly, one of the figures in the
second unit bears the same relationship to the figure represented by the question mark (?). You are therefore to locate the figure which
would replace the question mark (?) from the options [1], [2], [3] & [4].

IMS Learning Resources Pvt.Ltd.,Mumbai.All copyrights to this material vestswith IMS Learning Resources Pvt.Ltd.
No part of this materials either in part oras a whole shall be copied,printed,electronically reproduced,sold or distributed without the written
consent of IMS Learing Resources Pvt.Ltd.and any such violation would entail initiation of suitable legal proceedings.

Copyright

2)

[2]

3)

[3]

4)

[4]

Li
m

[1]

iv
at
e

1)

ite
d

Login ID:8FA03376/Student Name:ANSHULGARG/Overall Score:7

Pr

Explanation:

2)

[2]

3)

[3]

Le

[1]

IM

1)

ar

ni

ng

es

ou

rc

es

Question: 3
The second figure in the first unit of the problem figures bears a certain relationship to the first figure. Similarly, one of the figures in the
second unit bears the same relationship to the figure represented by the question mark (?). You are therefore to locate the figure which
would replace the question mark (?) from the options [1], [2], [3] & [4].

4)

[4]

Explanation:
The arrow head gets inverted and the other part remains on the same side attached to the tip. The angle of main line remains the same.
Hence, [1].
Question: 4
The second figure in the first unit of the problem figures bears a certain relationship to the first figure. Similarly, one of the figures in the
second unit bears the same relationship to the figure represented by the question mark (?). You are therefore to locate the figure which

IMS Learning Resources Pvt.Ltd.,Mumbai.All copyrights to this material vestswith IMS Learning Resources Pvt.Ltd.
No part of this materials either in part oras a whole shall be copied,printed,electronically reproduced,sold or distributed without the written
consent of IMS Learing Resources Pvt.Ltd.and any such violation would entail initiation of suitable legal proceedings.

Copyright

Login ID:8FA03376/Student Name:ANSHULGARG/Overall Score:7

[1]

2)

[2]

3)

[3]

4)

[4]

iv
at
e

1)

Li
m

ite
d

would replace the question mark (?) from the options [1], [2], [3] & [4].

Pr

Explanation:
The main object is the mirror image of the first object. The arrow tip on the right hand side of the box moves to the left hand side and the
arrow on the left hand side of the box gets replaced. Hence, [4].

2)

[2]

3)

[3]
[4]

IM

4)

Le

[1]

1)

ar

ni

ng

es

ou

rc

es

Question: 5
The second figure in the first unit of the problem figures bears a certain relationship to the first figure. Similarly, one of the figures in the
second unit bears the same relationship to the figure represented by the question mark (?). You are therefore to locate the figure which
would replace the question mark (?) from the options [1], [2], [3] & [4].

Explanation:

IMS Learning Resources Pvt.Ltd.,Mumbai.All copyrights to this material vestswith IMS Learning Resources Pvt.Ltd.
No part of this materials either in part oras a whole shall be copied,printed,electronically reproduced,sold or distributed without the written
consent of IMS Learing Resources Pvt.Ltd.and any such violation would entail initiation of suitable legal proceedings.

Copyright

ite
d

Login ID:8FA03376/Student Name:ANSHULGARG/Overall Score:7

[2]

3)

[3]

4)

[4]

rc

2)

ou

[1]

es

1)

es

Pr

iv
at
e

Li
m

Question: 6
In each of the questions given below, which one of the four answer figures below the problem figures should come after the problem
figures on the right, if the sequence were continued.

ng

Explanation:
Some elements move half a step and one and a half steps in alternate steps. New element introduced at every step in the centre of the
side (moving in the anticlockwise direction). Hence, [3].

IM

Le

ar

ni

Question: 7
In each of the questions given below, which one of the four answer figures below the problem figures should come after the problem
figures on the right, if the sequence were continued.

1)

[1]

IMS Learning Resources Pvt.Ltd.,Mumbai.All copyrights to this material vestswith IMS Learning Resources Pvt.Ltd.
No part of this materials either in part oras a whole shall be copied,printed,electronically reproduced,sold or distributed without the written
consent of IMS Learing Resources Pvt.Ltd.and any such violation would entail initiation of suitable legal proceedings.

Copyright

Login ID:8FA03376/Student Name:ANSHULGARG/Overall Score:7

2)

[2]

3)

[3]

4)

[4]

Li
m

ite
d

Explanation:

[2]

3)

[3]

4)

[4]

2)

ng

[1]

ni

1)

es

ou

rc

es

Pr

iv
at
e

Question: 8
In each of the questions given below, which one of the four answer figures below the problem figures should come after the problem
figures on the right, if the sequence were continued.

IM

Le

ar

Explanation:

Question: 9
In each of the questions given below, which one of the four answer figures below the problem figures should come after the problem
figures on the right, if the sequence were continued.

IMS Learning Resources Pvt.Ltd.,Mumbai.All copyrights to this material vestswith IMS Learning Resources Pvt.Ltd.
No part of this materials either in part oras a whole shall be copied,printed,electronically reproduced,sold or distributed without the written
consent of IMS Learing Resources Pvt.Ltd.and any such violation would entail initiation of suitable legal proceedings.

Copyright

2)

[2]

3)

[3]

4)

[4]

Li
m

[1]

iv
at
e

1)

ite
d

Login ID:8FA03376/Student Name:ANSHULGARG/Overall Score:7

Pr

Explanation:

[1]

2)

[2]

3)

[3]

4)

Le

1)

ar

ni

ng

es

ou

rc

es

Question: 10
In each of the questions given below, which one of the four answer figures below the problem figures should come after the problem
figures on the right, if the sequence were continued.

IM

[4]

Explanation:

Question: 11
In each of the questions given below, which one of the four answer figures below the problem figures should come after the problem

IMS Learning Resources Pvt.Ltd.,Mumbai.All copyrights to this material vestswith IMS Learning Resources Pvt.Ltd.
No part of this materials either in part oras a whole shall be copied,printed,electronically reproduced,sold or distributed without the written
consent of IMS Learing Resources Pvt.Ltd.and any such violation would entail initiation of suitable legal proceedings.

Copyright

Login ID:8FA03376/Student Name:ANSHULGARG/Overall Score:7

[1]

2)

[2]

3)

[3]

4)

[4]

iv
at
e

1)

Li
m

ite
d

figures on the right, if the sequence were continued.

es

ou

rc

es

Pr

Explanation:

IM

Le

ar

ni

ng

Question: 12
In each of the following problems, a related pair of figures is given followed by four numbered pairs of figures (1), (2), (3) & (4). Select a
pair that has a relationship similar to that in the original pair.

1)

[1]

2)

[2]

3)

[3]

4)

[4]

IMS Learning Resources Pvt.Ltd.,Mumbai.All copyrights to this material vestswith IMS Learning Resources Pvt.Ltd.
No part of this materials either in part oras a whole shall be copied,printed,electronically reproduced,sold or distributed without the written
consent of IMS Learing Resources Pvt.Ltd.and any such violation would entail initiation of suitable legal proceedings.

Copyright

Login ID:8FA03376/Student Name:ANSHULGARG/Overall Score:7

Explanation:
If the polygon in I has 'n' sides, then the polygon in II has 'n + 1' sides. Hence, [3].

[1]

2)

[2]

3)

[3]

4)

[4]

Pr

1)

iv
at
e

Li
m

ite
d

Question: 13
In each of the following problems, a related pair of figures is given followed by four numbered pairs of figures (1), (2), (3) & (4). Select a
pair that has a relationship similar to that in the original pair.

es

Explanation:
The IIndfigure is rotated 180 anticlockwise. Hence, [1].

2)

[2]

3)

[3]
[4]

IM

4)

Le

[1]

1)

ar

ni

ng

es

ou

rc

Question: 14
In each of the following problems, a related pair of figures is given followed by four numbered pairs of figures (1), (2), (3) & (4). Select a
pair that has a relationship similar to that in the original pair.

Explanation:
Figure II is a bigger version of figure I and is inverted. Hence, [4].
Question: 15
In each of the following problems, a related pair of figures is given followed by four numbered pairs of figures (1), (2), (3) & (4). Select a
pair that has a relationship similar to that in the original pair.

IMS Learning Resources Pvt.Ltd.,Mumbai.All copyrights to this material vestswith IMS Learning Resources Pvt.Ltd.
No part of this materials either in part oras a whole shall be copied,printed,electronically reproduced,sold or distributed without the written
consent of IMS Learing Resources Pvt.Ltd.and any such violation would entail initiation of suitable legal proceedings.

Copyright

2)

[2]

3)

[3]

4)

[4]

Li
m

[1]

Explanation:
The figure is rotated upside down and the shading is done in the other part. Hence, [4].

iv
at
e

1)

ite
d

Login ID:8FA03376/Student Name:ANSHULGARG/Overall Score:7

[2]

3)

[3]

4)

[4]

ng

2)

ni

[1]

ar

1)

es

ou

rc

es

Pr

Question: 16
In each of the following problems, a related pair of figures is given followed by four numbered pairs of figures (1), (2), (3) & (4). Select a
pair that has a relationship similar to that in the original pair.

Le

Explanation:
The figure inside is the figure I and the surrounding figure is the bigger invertedversion of the same figure. Hence, [2].

IM

Question: 17
Each question consists of a set of four figures split vertically into two. There is a definite relationship between the left-hand and right-hand
sections. Select the option in which the sections are not related to each other in the same way as in the other figures.

IMS Learning Resources Pvt.Ltd.,Mumbai.All copyrights to this material vestswith IMS Learning Resources Pvt.Ltd.
No part of this materials either in part oras a whole shall be copied,printed,electronically reproduced,sold or distributed without the written
consent of IMS Learing Resources Pvt.Ltd.and any such violation would entail initiation of suitable legal proceedings.

Copyright

[1]

2)

[2]

3)

[3]

4)

[4]

Li
m

1)

ite
d

Login ID:8FA03376/Student Name:ANSHULGARG/Overall Score:7

iv
at
e

Explanation:
In each of the options except [4], the mirror image of the first element takes the third position. The water image of the second element
takes the fourth position. The third element rotates 90 clockwise and takes the second position and the fourth element rotates 90
anticlockwise and takes the first position. Hence, [4].

2)

[2]

3)

[3]

4)

[4]

[1]

ng

1)

es

ou

rc

es

Pr

Question: 18
Each question consists of a set of four figures split vertically into two. There is a definite relationship between the left-hand and right-hand
sections. Select the option in which the sections are not related to each other in the same way as in the other figures.

ar

ni

Explanation:
The figure in the left half and its mirror image overlap to give the figure in the right half. Hence, [4].

IM

Le

Question: 19
Each question consists of a set of four figures split vertically into two. There is a definite relationship between the left-hand and right-hand
sections. Select the option in which the sections are not related to each other in the same way as in the other figures.

1)

[1]

IMS Learning Resources Pvt.Ltd.,Mumbai.All copyrights to this material vestswith IMS Learning Resources Pvt.Ltd.
No part of this materials either in part oras a whole shall be copied,printed,electronically reproduced,sold or distributed without the written
consent of IMS Learing Resources Pvt.Ltd.and any such violation would entail initiation of suitable legal proceedings.

Copyright

Login ID:8FA03376/Student Name:ANSHULGARG/Overall Score:7

2)

[2]

3)

[3]

4)

[4]

Explanation:
In each of the options, both the figures have similar elements, except [1] where one element is completely replaced. Hence, [1].

2)

[2]

3)

[3]

4)

[4]

Pr

[1]

es

1)

iv
at
e

Li
m

ite
d

Question: 20
Each question consists of a set of four figures split vertically into two. There is a definite relationship between the left-hand and right-hand
sections. Select the option in which the sections are not related to each other in the same way as in the other figures.

ou

rc

Explanation:
In all options except [3], the right half is obtained by eliminating the shaded section in the left half. Hence, [3].

[1]

2)

[2]
[3]

IM

3)

1)

Le

ar

ni

ng

es

Question: 21
Each question consists of a set of four figures split vertically into two. There is a definite relationship between the left-hand and right-hand
sections. Select the option in which the sections are not related to each other in the same way as in the other figures.

4)

[4]

Explanation:
The outer element rotates by 90oin clockwise direction and the inner element inverts laterally and then rotates 90oclockwise. Hence, [2].
Question: 22
Refer to the data below and answer the questions that follow.

IMS Learning Resources Pvt.Ltd.,Mumbai.All copyrights to this material vestswith IMS Learning Resources Pvt.Ltd.
No part of this materials either in part oras a whole shall be copied,printed,electronically reproduced,sold or distributed without the written
consent of IMS Learing Resources Pvt.Ltd.and any such violation would entail initiation of suitable legal proceedings.

Copyright

Login ID:8FA03376/Student Name:ANSHULGARG/Overall Score:7

Sujata

2)

Sunita

3)

Sumati

4)

Cannot be determined

Li
m

1)

ite
d

Who is the third tallest of them?

iv
at
e

Explanation:

Pr

Hence, [4].

2)

Sunita

3)

Sumati

4)

Sarita

ou

Sujata

es

1)

rc

Who is the third shortest of them?

es

Question: 23
Refer to the data below and answer the questions that follow.

ar

ni

ng

Explanation:

Le

Hence, [4].

Question: 24
Refer to the data below and answer the questions that follow.

IM

How many females are shorter than Sarita but taller than Sulekha?
1)

2)

3)

4)

Explanation:

IMS Learning Resources Pvt.Ltd.,Mumbai.All copyrights to this material vestswith IMS Learning Resources Pvt.Ltd.
No part of this materials either in part oras a whole shall be copied,printed,electronically reproduced,sold or distributed without the written
consent of IMS Learing Resources Pvt.Ltd.and any such violation would entail initiation of suitable legal proceedings.

Copyright

Login ID:8FA03376/Student Name:ANSHULGARG/Overall Score:7

Hence, [2].

ite
d

Question: 25
Refer to the data below and answer the questions that follow.
If Sujata is the tallest among all of them, how many females are shorter than Sunita but taller than Savita?
0

2)

3)

4)

Cannot be determined

iv
at
e

Li
m

1)

es

Pr

Explanation:

Le

ar

ni

ng

es

ou

Question: 26
Refer to the data below and answer the questions that follow.

rc

Hence, [4].

Who is married to Y?

IM

2)

1)

3)

4)

Either F or A

Explanation:

IMS Learning Resources Pvt.Ltd.,Mumbai.All copyrights to this material vestswith IMS Learning Resources Pvt.Ltd.
No part of this materials either in part oras a whole shall be copied,printed,electronically reproduced,sold or distributed without the written
consent of IMS Learing Resources Pvt.Ltd.and any such violation would entail initiation of suitable legal proceedings.

Copyright

es

ou

rc

es

Pr

iv
at
e

Li
m

ite
d

Login ID:8FA03376/Student Name:ANSHULGARG/Overall Score:7

ng

A is married to Y. Hence, [3].

ni

Question: 27
Refer to the data below and answer the questions that follow.

2)

3)

4)

Le

1)

ar

Who is/are definitely a child/children of F?

IM

Both E and H

Explanation:

IMS Learning Resources Pvt.Ltd.,Mumbai.All copyrights to this material vestswith IMS Learning Resources Pvt.Ltd.
No part of this materials either in part oras a whole shall be copied,printed,electronically reproduced,sold or distributed without the written
consent of IMS Learing Resources Pvt.Ltd.and any such violation would entail initiation of suitable legal proceedings.

Copyright

es

ou

rc

es

Pr

iv
at
e

Li
m

ite
d

Login ID:8FA03376/Student Name:ANSHULGARG/Overall Score:7

ng

C is the child of F. Hence, [3].

ni

Question: 28
Refer to the data below and answer the questions that follow.

2)

3)

4)

Le

1)

ar

Who amongst the following has 1 son and 1 daughter?

IM

None of these

Explanation:

IMS Learning Resources Pvt.Ltd.,Mumbai.All copyrights to this material vestswith IMS Learning Resources Pvt.Ltd.
No part of this materials either in part oras a whole shall be copied,printed,electronically reproduced,sold or distributed without the written
consent of IMS Learing Resources Pvt.Ltd.and any such violation would entail initiation of suitable legal proceedings.

Copyright

ng

F has 1 son and 1 daughter. Hence, [2].

es

ou

rc

es

Pr

iv
at
e

Li
m

ite
d

Login ID:8FA03376/Student Name:ANSHULGARG/Overall Score:7

ni

Question: 29
Refer to the data below and answer the questions that follow.

2)

F and Y

3)

B and A

4)

ar

F and A

Le

1)

If G is a female, then who definitely are the two children of D?

IM

None of these

Explanation:

IMS Learning Resources Pvt.Ltd.,Mumbai.All copyrights to this material vestswith IMS Learning Resources Pvt.Ltd.
No part of this materials either in part oras a whole shall be copied,printed,electronically reproduced,sold or distributed without the written
consent of IMS Learing Resources Pvt.Ltd.and any such violation would entail initiation of suitable legal proceedings.

Copyright

es

ou

rc

es

Pr

iv
at
e

Li
m

ite
d

Login ID:8FA03376/Student Name:ANSHULGARG/Overall Score:7

ng

If G is a female, the F and A are the two children of D. Hence, [1].

ni

Question: 30
Refer to the data below and answer the questions that follow.

2)

3)

4)

Le

1)

ar

If G is a male, then who is definitely son of X?

IM

None of these

Explanation:

IMS Learning Resources Pvt.Ltd.,Mumbai.All copyrights to this material vestswith IMS Learning Resources Pvt.Ltd.
No part of this materials either in part oras a whole shall be copied,printed,electronically reproduced,sold or distributed without the written
consent of IMS Learing Resources Pvt.Ltd.and any such violation would entail initiation of suitable legal proceedings.

Copyright

es

ou

rc

es

Pr

iv
at
e

Li
m

ite
d

Login ID:8FA03376/Student Name:ANSHULGARG/Overall Score:7

ng

If G is a male, either F or A can be son of X. Hence, [4].

ni

Question: 31
Refer to the data below and answer the questions that follow.

paternal grandfather

2)

paternal grandmother

3)

maternal grandfather

4)

Le

1)

ar

For the previous question, if we add one more condition that F is not the sibling of Y, then X will be the ________________ of C.

IM

None of these

Explanation:

IMS Learning Resources Pvt.Ltd.,Mumbai.All copyrights to this material vestswith IMS Learning Resources Pvt.Ltd.
No part of this materials either in part oras a whole shall be copied,printed,electronically reproduced,sold or distributed without the written
consent of IMS Learing Resources Pvt.Ltd.and any such violation would entail initiation of suitable legal proceedings.

Copyright

es

ou

rc

es

Pr

iv
at
e

Li
m

ite
d

Login ID:8FA03376/Student Name:ANSHULGARG/Overall Score:7

ng

If F is not sibling of Y, then A and B will be brother-sister pair and X will be maternal grandfather of C. Hence, [3].

ni

Question: 32
Read the following short passage and answer the question that follows.

1)

Mumbai's police commissioner for traffic should be chastised severely.

2)

The traffic police should take incidents of drunk driving more seriously.

3)

Le

ar

Mumbai has been experiencing an unexpected increase in traffic accidents in the past year.
What is the best course of action to be followed in the above situation?

IM

Traffic police in Mumbai should be better trained to respond to accidents.

4)

The public in Mumbai should be better educated about the rules of road safety.

Explanation:
Chastising the police commissioner will not change the fact that the accidents occurred, nor will it necessarily help prevent them in the
future, so [1] is not a particularly helpful course of action. [3] can be eliminated for the same reasons. While [2] may seem like a
reasonable course of action at first, there is no suggestion in the passage that the accident rate increased only or mainly due to incidents
of drunk driving; there may be many other factors involved. A more general preventative measure would be to educate people about
driving responsibly (which would include not driving while drunk). Hence, [4].

IMS Learning Resources Pvt.Ltd.,Mumbai.All copyrights to this material vestswith IMS Learning Resources Pvt.Ltd.
No part of this materials either in part oras a whole shall be copied,printed,electronically reproduced,sold or distributed without the written
consent of IMS Learing Resources Pvt.Ltd.and any such violation would entail initiation of suitable legal proceedings.

Copyright

Login ID:8FA03376/Student Name:ANSHULGARG/Overall Score:7

Question: 33
Read the following short passage and answer the question that follows.
Even after hearing on the news that a fire had broken out in the office building where her husband worked, Roshni was completely
unconcerned and went about her day without worrying.
Which of the following, if true, best explains Roshni's behaviour?
Roshni was aware that the fire was not very serious and did not spread to all the floors of the building.

2)

Roshni was not the type of person to worry needlessly about things she had no control over.

3)

Roshni did not care about her husband very much.

4)

Roshni knew that her husband was on a business trip in another city on that day.

Li
m

ite
d

1)

iv
at
e

Explanation:
It is human nature to worry about one's spouse in such a life-threatening situation, even if the danger involved is minimal, so [1] is not a
good explanation. Most people worry especially when they have no control over dangerous situations, so [2] does not seem like a very
realistic explanation. While [3] may seem plausible, it makes Roshni seem like a psychopath, so it should be taken with a grain of salt. The
best explanation is [4]: if Roshni knew that her husband would not be in his office on that day, then he would be in no danger from the fire,
and therefore she would have no cause to worry. Hence, [4].

Only b

3)

Either a or b

4)

Both a & b

2)

ng

Only a

ni

1)

es

ou

rc

es

Pr

Question: 34
The statement below is followed by two courses of action. On the basis of the information given in the statement, choose the most
appropriate course of action.

Le

ar

Explanation:
It is not a good idea to enforce a complete ban, since non-infected people would unnecessarily be inconvenienced. The most appropriate
course of action is to take appropriate precautionary measures by setting up detection centres to screen the passengers and isolate the
infected ones immediately. Hence, [2].

IM

Question: 35
Refer to the data below and answer the questions that follow.

IMS Learning Resources Pvt.Ltd.,Mumbai.All copyrights to this material vestswith IMS Learning Resources Pvt.Ltd.
No part of this materials either in part oras a whole shall be copied,printed,electronically reproduced,sold or distributed without the written
consent of IMS Learing Resources Pvt.Ltd.and any such violation would entail initiation of suitable legal proceedings.

Copyright

Li
m

ite
d

Login ID:8FA03376/Student Name:ANSHULGARG/Overall Score:7

1)

Anil

2)

Mukesh

3)

Dinesh

4)

Cannot be detrmined

iv
at
e

Who has got the second highest income?

ar

ni

ng

es

ou

rc

es

Pr

Explanation:

Le

Hence, [2].

Question: 36
Refer to the data below and answer the questions that follow.

IM

Who has got the least income?


1)

Anil

2)

Mahesh

3)

Dinesh

4)

Mukesh

Explanation:

IMS Learning Resources Pvt.Ltd.,Mumbai.All copyrights to this material vestswith IMS Learning Resources Pvt.Ltd.
No part of this materials either in part oras a whole shall be copied,printed,electronically reproduced,sold or distributed without the written
consent of IMS Learing Resources Pvt.Ltd.and any such violation would entail initiation of suitable legal proceedings.

Copyright

iv
at
e

Li
m

ite
d

Login ID:8FA03376/Student Name:ANSHULGARG/Overall Score:7

Hence, [2].

Pr

Question: 37
Refer to the data below and answer the questions that follow.
What is the income of Dinesh?
Rs.12

2)

Rs.35

3)

Rs.47

4)

None of these

ou

rc

es

1)

IM

Le

ar

ni

ng

es

Explanation:

Hence, [2].
Question: 38
Refer to the data below and answer the questions that follow.

IMS Learning Resources Pvt.Ltd.,Mumbai.All copyrights to this material vestswith IMS Learning Resources Pvt.Ltd.
No part of this materials either in part oras a whole shall be copied,printed,electronically reproduced,sold or distributed without the written
consent of IMS Learing Resources Pvt.Ltd.and any such violation would entail initiation of suitable legal proceedings.

Copyright

Login ID:8FA03376/Student Name:ANSHULGARG/Overall Score:7

Who is the person from Pune?


1)

Mahesh

2)

Mukesh

3)

Rahul

4)

Dinesh

es

Pr

iv
at
e

Li
m

ite
d

Explanation:

es

ou

Question: 39
Refer to the data below and answer the questions that follow.

rc

Hence, [4].

2)

Rs.70

3)

Rs.110

4)

Rs.98

ng

Rs.82

ni

1)

What is the combined income of the two highest paid persons in the group?

IM

Le

ar

Explanation:

IMS Learning Resources Pvt.Ltd.,Mumbai.All copyrights to this material vestswith IMS Learning Resources Pvt.Ltd.
No part of this materials either in part oras a whole shall be copied,printed,electronically reproduced,sold or distributed without the written
consent of IMS Learing Resources Pvt.Ltd.and any such violation would entail initiation of suitable legal proceedings.

Copyright

iv
at
e

Li
m

ite
d

Login ID:8FA03376/Student Name:ANSHULGARG/Overall Score:7

Hence, [3].

Pr

Question: 40
Find the odd man out from the series.

2)

187

3)

377

4)

761

rc

92

ou

1)

es

45, 92, 187, 377, 761, 1528

ni

ng

es

Explanation:

Le

ar

Question: 41
Find the odd man out from the series.

1)

10

2)

10, 17, 12, 18, 15, 19, 16, 20

IM

12

3)

18

4)

15

Explanation:
The above series consists of 2 alternate series one consisting of 10, 12, 14, 16 and second being 17, 18, 19, 20. So in the 1st series 15 is
wrong and instead of it 14 should be in its place. Hence, [4].
Question: 42

IMS Learning Resources Pvt.Ltd.,Mumbai.All copyrights to this material vestswith IMS Learning Resources Pvt.Ltd.
No part of this materials either in part oras a whole shall be copied,printed,electronically reproduced,sold or distributed without the written
consent of IMS Learing Resources Pvt.Ltd.and any such violation would entail initiation of suitable legal proceedings.

Copyright

Login ID:8FA03376/Student Name:ANSHULGARG/Overall Score:7

Find the odd man out from the series.


0, 4, 16,48, 100, 180
1)

2)

16

3)

48

4)

100

Li
m

ite
d

Explanation:

iv
at
e

Question: 43
Choose the correct alternative.

Five friends have a different sum with themselves Rahul's amount is less than the amount with Soham, Mridul has the highest amount
among them all while Guru is not the person with the least amount. Sumedh has an amount which is greater than Soham but less than
Mridul, who is the person with the second least amount with them?
Rahul

2)

Guru

3)

Soham

4)

Cannot be determined.

rc

es

Pr

1)

ar

ni

ng

es

ou

Explanation:

IM

Le

Question: 44
Each question has some statements followed by a set of conclusions. Choose the conclusions that logically follow from the given
statements.

1)

Only a

2)

Only b

3)

Either a or b

4)

Neither a nor b

IMS Learning Resources Pvt.Ltd.,Mumbai.All copyrights to this material vestswith IMS Learning Resources Pvt.Ltd.
No part of this materials either in part oras a whole shall be copied,printed,electronically reproduced,sold or distributed without the written
consent of IMS Learing Resources Pvt.Ltd.and any such violation would entail initiation of suitable legal proceedings.

Copyright

Login ID:8FA03376/Student Name:ANSHULGARG/Overall Score:7

ite
d

Explanation:

iv
at
e

Li
m

Question: 45
Each question has some statements followed by a set of conclusions. Choose the conclusions that logically follow from the given
statements.

Only [a]

2)

Only [b]

3)

Either [a] or [b]

4)

Both [a] and [b]

rc

es

Pr

1)

IM

Le

ar

ni

ng

es

ou

Explanation:

Question: 46
Refer to the sequence below and answer the questions that follow.
9CBP6A4DM3JKYX1REZ87NS
Which is the 3rdelement to the right of the 8thelement conunted from the extreme right end of the sequence?
1)

2)

IMS Learning Resources Pvt.Ltd.,Mumbai.All copyrights to this material vestswith IMS Learning Resources Pvt.Ltd.
No part of this materials either in part oras a whole shall be copied,printed,electronically reproduced,sold or distributed without the written
consent of IMS Learing Resources Pvt.Ltd.and any such violation would entail initiation of suitable legal proceedings.

Copyright

Login ID:8FA03376/Student Name:ANSHULGARG/Overall Score:7

3)

4)

Explanation:
The third element to the right of eighth element from the right is the (8 3 = 5) fifth element from the right of the sequence i.e., Z. Hence,
[3].

ite
d

Question: 47
Refer to the sequence below and answer the questions that follow.

2)

3)

4)

iv
at
e

1)

Li
m

9CBP6A4DM3JKYX1REZ87NS
If all the numbers are skipped from the series, then what will be the 4thletter to the left of 5thletter from the extreme left end in the new
sequence?

rc

ou

Question: 48
Refer to the sequence below and answer the questions that follow.

es

Pr

Explanation:

2)

3)

4)

ng

1)

es

9CBP6A4DM3JKYX1REZ87NS
How many numbers in the series are immediately followed or preceded by a vowel?

ar

ni

Explanation:
6A and A4 are two such cases. Hence, [3].

Le

Question: 49
Refer to the sequence below and answer the questions that follow.

IM

9CBP6A4DM3JKYX1REZ87NS
Select the odd man out from the given sequence: 9C P6 DM X1 87.
1)

9C

2)

P6

3)

DM

4)

X1

Explanation:

IMS Learning Resources Pvt.Ltd.,Mumbai.All copyrights to this material vestswith IMS Learning Resources Pvt.Ltd.
No part of this materials either in part oras a whole shall be copied,printed,electronically reproduced,sold or distributed without the written
consent of IMS Learing Resources Pvt.Ltd.and any such violation would entail initiation of suitable legal proceedings.

Copyright

Login ID:8FA03376/Student Name:ANSHULGARG/Overall Score:7

Question: 50
Refer to the sequence below and answer the questions that follow.

2)

3)

4)

Li
m

1)

ite
d

9CBP6A4DM3JKYX1REZ87NS
Which of the options will come in the place of the question mark based on the below sequence?
C B 6 4 J Y (?)

iv
at
e

Explanation:
The elements occupy the prime position. 2, 3, 5, 7, 11, 13, 17. Hence, the next term is the term in 17thposition. Hence, [4].

ni

ng

es

ou

rc

es

Pr

Question: 51
Six sentences are given below. Rearrange them to form a coherent passage and answer the questions that follow.

II

2)

III

3)

IV

4)

Le

1)

ar

Which of these should be the last sentence after rearrangement?

IM

VI

Explanation:
The first sentence I should be the opening sentence as it introduces the meaning of euthanasia. IV should come next as it clarifies why
euthanasia is considered a homicide. III should come next as it explains that not all types of homicides are considered unlawful and it
gives the types that aren't. Then II goes on to clarify that these categories are not used for euthanasia. V should come next as both II and
V talk of physician-assisted suicide. VI is the last sentence of the paragraph as it moves away from physician-assisted suicide to the kind
of life-ending measures that are considered legal almost everywhere in the US. Thus, the correct sequence is I-IV-III-II-V-VI. Hence, [4].
Question: 52

IMS Learning Resources Pvt.Ltd.,Mumbai.All copyrights to this material vestswith IMS Learning Resources Pvt.Ltd.
No part of this materials either in part oras a whole shall be copied,printed,electronically reproduced,sold or distributed without the written
consent of IMS Learing Resources Pvt.Ltd.and any such violation would entail initiation of suitable legal proceedings.

Copyright

Login ID:8FA03376/Student Name:ANSHULGARG/Overall Score:7

Six sentences are given below. Rearrange them to form a coherent passage and answer the questions that follow.

1)

2)

III

3)

IV

4)

Explanation:
Hence, [4].

Li
m

Question: 53
Six sentences are given below. Rearrange them to form a coherent passage and answer the questions that follow.

2)

II

3)

III

4)

IV

Pr

iv
at
e

Which of these should be the first sentence after rearrangement?


1)

ite
d

Which sentence should immediately follow II?

es

Explanation:
Hence, [1].

ou

rc

Question: 54
Six sentences are given below. Rearrange them to form a coherent passage and answer the questions that follow.

2)

II

3)

III

4)

IV

ng

1)

es

Which of these should be the third sentence after rearrangement?

ni

Explanation:
Hence, [3].

Le

ar

Question: 55
Six sentences are given below. Rearrange them to form a coherent passage and answer the questions that follow.

1)

IV

2)

The removal of which of the following sentences (only one at a time) would not break the flow of the paragraph?

IM

3)

VI

4)

Any of the above

Explanation:
If we remove sentences IV, V and VI one at a time from the paragraph, we see that the flow of the paragraph is not affected. E.g., if we
remove IV, we are left with the following completely logically flowing paragraph: 'West's Encyclopedia of American Law states that 'a
"mercy killing" or euthanasia is generally considered to be a criminal homicide' and is normally used as a synonym of homicide committed
at a request made by the patient. Not all homicide is unlawful however: the two designations of homicide that carry no criminal punishment

IMS Learning Resources Pvt.Ltd.,Mumbai.All copyrights to this material vestswith IMS Learning Resources Pvt.Ltd.
No part of this materials either in part oras a whole shall be copied,printed,electronically reproduced,sold or distributed without the written
consent of IMS Learing Resources Pvt.Ltd.and any such violation would entail initiation of suitable legal proceedings.

Copyright

Login ID:8FA03376/Student Name:ANSHULGARG/Overall Score:7

are 'justifiable' and 'excusable' homicide, but these classifications have not been extended to euthanasia. The term 'euthanasia' is usually
confined to the 'active' variety; the University of Washington website states that 'euthanasia generally means that the physician would act
directly, for instance by giving a lethal injection, to end the patient's life'. Physician-assisted suicide is thus not classified as euthanasia by
the US State of Oregon, where euthanasia is actually legal under the Oregon Death with Dignity Act, and despite its name, it is not legally
classified as suicide either. Unlike physician-assisted suicide, withholding or withdrawing life-sustaining treatments with patient consent
('voluntary') is almost unanimously considered, at least in the United States, to be legal.' Similarly, if we remove only V and then only VI,
the remaining paragraph continues to maintain a logical flow. Hence, [4].

Only [a]

2)

Only [b]

3)

Either [a] or [b]

4)

Neither [a] nor [b]

Pr

1)

iv
at
e

Li
m

ite
d

Question: 56
Each question has some statements followed by a set of conclusions. Choose the conclusions that logically follow from the given
statements.

ni

ng

es

ou

rc

es

Explanation:

IM

Le

ar

Question: 57
Each question has some statements followed by a set of conclusions. Choose the conclusions that logically follow from the given
statements.

1)

Only [a]

2)

Only [b]

3)

Either [a] or [b]

4)

Neither [a] nor [b]

IMS Learning Resources Pvt.Ltd.,Mumbai.All copyrights to this material vestswith IMS Learning Resources Pvt.Ltd.
No part of this materials either in part oras a whole shall be copied,printed,electronically reproduced,sold or distributed without the written
consent of IMS Learing Resources Pvt.Ltd.and any such violation would entail initiation of suitable legal proceedings.

Copyright

Login ID:8FA03376/Student Name:ANSHULGARG/Overall Score:7

ite
d

Explanation:

Only [a]

2)

Only [b]

3)

Either [a] or [b]

4)

Neither [a] nor [b]

es

1)

Pr

iv
at
e

Li
m

Question: 58
Each question has some statements followed by a set of conclusions. Choose the conclusions that logically follow from the given
statements.

Le

ar

ni

ng

es

ou

rc

Explanation:

IM

Question: 59
Each question has some statements followed by a set of conclusions. Choose the conclusions that logically follow from the given
statements.

IMS Learning Resources Pvt.Ltd.,Mumbai.All copyrights to this material vestswith IMS Learning Resources Pvt.Ltd.
No part of this materials either in part oras a whole shall be copied,printed,electronically reproduced,sold or distributed without the written
consent of IMS Learing Resources Pvt.Ltd.and any such violation would entail initiation of suitable legal proceedings.

Copyright

1)

Only [a]

2)

Only [b]

3)

Either [a] or [b]

4)

Neither [a] nor [b]

ite
d

Login ID:8FA03376/Student Name:ANSHULGARG/Overall Score:7

es

Pr

iv
at
e

Li
m

Explanation:

2)

Only [b]

3)

Either [a] or [b]

4)

Neither [a] nor [b]

ni

Only [a]

ar

1)

ng

es

ou

rc

Question: 60
Each question has some statements followed by a set of conclusions. Choose the conclusions that logically follow from the given
statements.

Le

Explanation:
Nothing can be said for sure about statements with uncertain premises. An actor may/may not end up being a dancer and a dancer
may/may not end up being talented. Hence, [4].

IM

Question: 61
Refer to the data below and answer the questions that follow.

What is the code for K?

IMS Learning Resources Pvt.Ltd.,Mumbai.All copyrights to this material vestswith IMS Learning Resources Pvt.Ltd.
No part of this materials either in part oras a whole shall be copied,printed,electronically reproduced,sold or distributed without the written
consent of IMS Learing Resources Pvt.Ltd.and any such violation would entail initiation of suitable legal proceedings.

Copyright

Login ID:8FA03376/Student Name:ANSHULGARG/Overall Score:7

1)

2)

3)

4)

None of these

ite
d

Explanation:
Looking at words 'EDGE', E is coded as 9. Also looking at 'EDGE' and 'FED', D is coded as 7. So G will be coded as 5 and F as 8. K will
be coded as 1. Codes for O and R are 3 and 4 (not necessarily in that order) and codes for N and I will be 2 and 6 (not necessarily in that
order).
Code for K is 1. Hence, [1].

Li
m

Question: 62
Refer to the data below and answer the questions that follow.

2)

3)

4)

None of these

Pr

1)

iv
at
e

7 is the code for

ou

rc

es

Explanation:
Looking at words 'EDGE', E is coded as 9. Also looking at 'EDGE' and 'FED', D is coded as 7. So G will be coded as 5 and F as 8. K will
be coded as 1. Codes for O and R are 3 and 4 (not necessarily in that order) and codes for N and I will be 2 and 6 (not necessarily in that
order)
7 is code of D. Hence, [3].

es

Question: 63
Refer to the data below and answer the questions that follow.

2)

3)

4)

ng

ni

1)

Difference in values of the numerical code for F and I can be:

Le

ar

Explanation:
Looking at words 'EDGE', E is coded as 9. Also looking at 'EDGE' and 'FED', D is coded as 7. So G will be coded as 5 and F as 8. K will
be coded as 1. Codes for O and R are 3 and 4 (not necessarily in that order) and codes for N and I will be 2 and 6 (not necessarily in that
order)
Difference in code F and I can be 2 or 6. Hence, [4].

IM

Question: 64
Refer to the data below and answer the questions that follow.
'RIDE' can be coded as each of the following except:
1)

3679

2)

2679

3)

2479

IMS Learning Resources Pvt.Ltd.,Mumbai.All copyrights to this material vestswith IMS Learning Resources Pvt.Ltd.
No part of this materials either in part oras a whole shall be copied,printed,electronically reproduced,sold or distributed without the written
consent of IMS Learing Resources Pvt.Ltd.and any such violation would entail initiation of suitable legal proceedings.

Copyright

Login ID:8FA03376/Student Name:ANSHULGARG/Overall Score:7

4)

4679

Explanation:
Looking at words 'EDGE', E is coded as 9. Also looking at 'EDGE' and 'FED', D is coded as 7. So G will be coded as 5 and F as 8. K will
be coded as 1. Codes for O and R are 3 and 4 (not necessarily in that order) and codes for N and I will be 2 and 6 (not necessarily in that
order)
Both 2 and 6 together cannot be part of the code. Hence, [2].

ite
d

Question: 65
Refer to the data below and answer the questions that follow.
'1267' can be the code for:
RODE

2)

KIND

3)

FORD

4)

None of these

iv
at
e

Li
m

1)

Pr

Explanation:
Looking at words 'EDGE', E is coded as 9. Also looking at 'EDGE' and 'FED', D is coded as 7. So G will be coded as 5 and F as 8. K will
be coded as 1. Codes for O and R are 3 and 4 (not necessarily in that order) and codes for N and I will be 2 and 6 (not necessarily in that
order)
Only possible word is KIND. Hence, [2].

rc

es

Question: 66
Refer to the data below and answer the questions that follow.

ou

If code for Y is 0 then KIDNEY will be coded as:


012457

2)

013467

3)

012567

4)

None of these

es

1)

ar

ni

ng

Explanation:
Looking at words 'EDGE', E is coded as 9. Also looking at 'EDGE' and 'FED', D is coded as 7. So G will be coded as 5 and F as 8. K will
be coded as 1. Codes for O and R are 3 and 4 (not necessarily in that order) and codes for N and I will be 2 and 6 (not necessarily in that
order)
Code for KIDNEY will be 012679. Hence, [4].

IM

Le

Question: 67
Refer to the data below and answer the questions that follow.

IMS Learning Resources Pvt.Ltd.,Mumbai.All copyrights to this material vestswith IMS Learning Resources Pvt.Ltd.
No part of this materials either in part oras a whole shall be copied,printed,electronically reproduced,sold or distributed without the written
consent of IMS Learing Resources Pvt.Ltd.and any such violation would entail initiation of suitable legal proceedings.

Copyright

Li
m

ite
d

Login ID:8FA03376/Student Name:ANSHULGARG/Overall Score:7

1)

2)

3)

4)

Cannot be determined

iv
at
e

What is the number on the sector which is opposite to the sector with number 3?

IM

Le

ar

ni

ng

es

ou

rc

es

Pr

Explanation:

The number on the sector opposite to the sector with number 3 on it is 2.

IMS Learning Resources Pvt.Ltd.,Mumbai.All copyrights to this material vestswith IMS Learning Resources Pvt.Ltd.
No part of this materials either in part oras a whole shall be copied,printed,electronically reproduced,sold or distributed without the written
consent of IMS Learing Resources Pvt.Ltd.and any such violation would entail initiation of suitable legal proceedings.

Copyright

Login ID:8FA03376/Student Name:ANSHULGARG/Overall Score:7

Hence, [1].

1)

2)

3)

4)

Cannot be determined

ite
d

Question: 68
Refer to the data below and answer the questions that follow.
If Cathy gets a score of 6, Danny gets a score of:

Le

ar

ni

ng

es

ou

rc

es

Pr

iv
at
e

Li
m

Explanation:

IM

If Cathy gets a score of 6, Cathy and Bill's scores are on the either sides of the number 3. Hence, Danny's score is the number on a sector
between the sectors which have the numbers that are the scores of Ann and Fenny. Danny's score is 4 or 5. Hence, [4].
Question: 69
Refer to the data below and answer the questions that follow.
If Don gets a score of 5 and Bill hits a sector between the sectors hit by Jill and Ron, then the number on the sector opposite to the sector
with 5 on it is:
1)

2)

IMS Learning Resources Pvt.Ltd.,Mumbai.All copyrights to this material vestswith IMS Learning Resources Pvt.Ltd.
No part of this materials either in part oras a whole shall be copied,printed,electronically reproduced,sold or distributed without the written
consent of IMS Learing Resources Pvt.Ltd.and any such violation would entail initiation of suitable legal proceedings.

Copyright

Login ID:8FA03376/Student Name:ANSHULGARG/Overall Score:7

3)

4)

es

ou

rc

es

Pr

iv
at
e

Li
m

ite
d

Explanation:

ni

ng

Bill's score is between 3 and 8 i.e., 7 is between the numbers 3 and 8.


The number on the sector opposite the sector with number 5 (i.e., the score of Don) is 6. Hence, [2].

1)

2)

3)

Le

ar

Question: 70
Refer to the data below and answer the questions that follow.
If Don gets a score of 4 and Cathy gets a score of 5, what is Danny's score?

IM

4)

Cannot be determined

Explanation:

IMS Learning Resources Pvt.Ltd.,Mumbai.All copyrights to this material vestswith IMS Learning Resources Pvt.Ltd.
No part of this materials either in part oras a whole shall be copied,printed,electronically reproduced,sold or distributed without the written
consent of IMS Learing Resources Pvt.Ltd.and any such violation would entail initiation of suitable legal proceedings.

Copyright

es

ou

rc

es

Pr

iv
at
e

Li
m

ite
d

Login ID:8FA03376/Student Name:ANSHULGARG/Overall Score:7

If Don = 4 and Cathy = 5, then Danny = 6. Hence, [3].

3)

4)

IM

Explanation:

ni

2)

ar

Le

1)

ng

Question: 71
Refer to the data below and answer the questions that follow.
If the sector with the number 5 on it is adjacent to the sector with the number 8 on it, then what is Don's score?

IMS Learning Resources Pvt.Ltd.,Mumbai.All copyrights to this material vestswith IMS Learning Resources Pvt.Ltd.
No part of this materials either in part oras a whole shall be copied,printed,electronically reproduced,sold or distributed without the written
consent of IMS Learing Resources Pvt.Ltd.and any such violation would entail initiation of suitable legal proceedings.

Copyright

es

ou

rc

es

Pr

iv
at
e

Li
m

ite
d

Login ID:8FA03376/Student Name:ANSHULGARG/Overall Score:7

If 5 is adjacent to 8, then Don's score is 5. Hence, [1].

ng

Question: 72
Read the following short passage and answer the question that follows.

Le

ar

ni

The potato's origins are firmly rooted in the Andes. The ancestral species grow wild there hundreds of scraggly, undistinguished plants
whose value as food is far from obvious. The foliage cannot be eaten because it is packed with poisonous glyco-alkaloids; the tubers of
most species are poisonous too, and small, so that it is difficult to imagine what might have initially encouraged people to experiment with
them. Yet experiment they must have done, and archaeological evidence indicates that the process that ultimately produced edible
cultivated potatoes began more than 8,000 years ago.
Which of the following, if true, best explains why the ancient people of the Andes started experimenting with the wild ancestral species of
potatoes?

Ancestral wild potatoes were easy to cultivate, so the ancient Andeans took to growing them in large numbers.

IM

1)
2)

The ancient Andeans were careful not to choose the poisonous species of ancestral wild potatoes for their experiments.

3)

A large-scale famine around 8 millennia ago forced the ancient Andeans to try eating any plant available, whether or not it
was known to be edible.

4)

The ancient Andeans initially cultivated wild potatoes as ornamental plants, and only later started eating them after
discovering they were edible.

Explanation:
[1] does not explain why the ancient Andeans started growing potatoes if most of them were poisonous. [2] makes no sense: how did the

IMS Learning Resources Pvt.Ltd.,Mumbai.All copyrights to this material vestswith IMS Learning Resources Pvt.Ltd.
No part of this materials either in part oras a whole shall be copied,printed,electronically reproduced,sold or distributed without the written
consent of IMS Learing Resources Pvt.Ltd.and any such violation would entail initiation of suitable legal proceedings.

Copyright

Login ID:8FA03376/Student Name:ANSHULGARG/Overall Score:7

ancient Andeans know which species were poisonous without experimenting with them first? [4] does not resolve the issue, as the
question as to why the ancient Andeans started eating the potatoes still remains. [3] is the best explanation: if the ancient Andeans were
desperate for food, they may have started eating potatoes without knowing which ones were poisonous, and thus inadvertently discovered
the few species that weren't poisonous.
Hence, [3].

rc

es

Pr

iv
at
e

Li
m

ite
d

Question: 73
Read the following information followed by six statements carefully and answer the questions that follow.

2)

[b]

3)

[c]

4)

[e]

es

[a]

1)

ou

Which of the following can be concluded from the given paragraph?

Le

ar

ni

ng

Explanation:
[a] involves a misunderstanding of the passage: lifestyle changes cause changes in how one's genes are expressed, not in the genes
themselves. [b] assumes information not given in the paragraph, i.e. what exactly constitutes 'nature' and 'nurture'. [c] is a possible
outcome or inference drawn from the given information, not the conclusion of the whole argument. Only [e] is a suitable conclusion: the
point of the argument is to show how gene expression can be significantly altered by lifestyle changes.
Hence, [4].

IM

Question: 74
Read the following information followed by six statements carefully and answer the questions that follow.
Which of the following could be a logical outcome of the information presented in the given paragraph?
1)

[a]

2)

[c]

3)

[d]

4)

[e]

Explanation:
[a] and [e] are not outcomes, so they can be ruled out at once. [d] is rather vague: what 'poor' genes are, and the 'sacrifices' involved, are

IMS Learning Resources Pvt.Ltd.,Mumbai.All copyrights to this material vestswith IMS Learning Resources Pvt.Ltd.
No part of this materials either in part oras a whole shall be copied,printed,electronically reproduced,sold or distributed without the written
consent of IMS Learing Resources Pvt.Ltd.and any such violation would entail initiation of suitable legal proceedings.

Copyright

Login ID:8FA03376/Student Name:ANSHULGARG/Overall Score:7

not explained. Only [c] is an outcome of the information given in the paragraph: according to the last sentence, the lifestyle changes
mentioned in the paragraph can change the part of our chromosomes that control how long we live. Hence, [2].
Question: 75
Choose the correct alternative.
Amit decides to take his car from his home to office. He sets out for office straight, but then realizes his car has malfunctioned. His car
takes or series of 5 right turns and then a series of 4 left turns. He observes that he is facing in south east direction. Towards which
direction did he start from home?
Northwest

2)

Southeast

3)

Northeast

4)

None of these

Li
m

ite
d

1)

Le

ar

Question: 76
Choose the correct alternative.

ni

ng

es

ou

rc

es

Pr

iv
at
e

Explanation:

25 km, northeast

IM

1)

Rani decides to take tour of the city on her scooty. She sets off towards east direction, covers 10 km and takes a right. She now covers 5
km and then goes towards west direction. She covers 34 km in this direction and takes a right and finally stops after 12 km. How far is she
from starting point and what is direction of and point starting point?

2)

10 km, northeast

3)

25 km, northwest

4)

Cannot be determined

Explanation:

IMS Learning Resources Pvt.Ltd.,Mumbai.All copyrights to this material vestswith IMS Learning Resources Pvt.Ltd.
No part of this materials either in part oras a whole shall be copied,printed,electronically reproduced,sold or distributed without the written
consent of IMS Learing Resources Pvt.Ltd.and any such violation would entail initiation of suitable legal proceedings.

Copyright

Li
m

ite
d

Login ID:8FA03376/Student Name:ANSHULGARG/Overall Score:7

iv
at
e

Question: 77
Choose the correct alternative.

Y is to the south of X, who is to the north of Z. M is to the south west of X. What is the direction of M with respect to Y?
Northwest

2)

Southwest

3)

South

4)

Cannot be determined

es

Pr

1)

ng

es

ou

rc

Explanation:

ar

Question: 78
Choose the correct alternative.

ni

Three possibilities arise as for as position of M is concerned. Hence direction of M with respect to Y cannot be determined. Hence, [4].

1)

North

2)

Le

Ramesh walks in a certain direction for 12km then takes a right turn and walks 5km; takes a left turn and walks for 6km; takes another left
turn and walks in the South West direction. Which direction did Ramesh initially start walking in?

3)

North West

4)

Cannot be determined

IM

North East

Explanation:

IMS Learning Resources Pvt.Ltd.,Mumbai.All copyrights to this material vestswith IMS Learning Resources Pvt.Ltd.
No part of this materials either in part oras a whole shall be copied,printed,electronically reproduced,sold or distributed without the written
consent of IMS Learing Resources Pvt.Ltd.and any such violation would entail initiation of suitable legal proceedings.

Copyright

Pr

iv
at
e

Li
m

ite
d

Login ID:8FA03376/Student Name:ANSHULGARG/Overall Score:7

rc

es

Question: 79
Choose the correct alternative.

ou

There are two rows of students seated in a class. Pramod observes that Praful sits exactly to the south of Rosy who sits to the North East
of Mahesh. Mahesh sits to the West of Pramod. In which direction with respect to Rosy is Pramod sitting?
South

2)

South East

3)

South West

4)

Cannot be determined

Le

ar

ni

ng

Explanation:

es

1)

IM

Question: 80
Choose the correct alternative.
How many pair of letters exist in the word 'FORESIGHT' which have as many letters between them in the word given above as the
number of letters between them in the English alphabet?
1)

2)

IMS Learning Resources Pvt.Ltd.,Mumbai.All copyrights to this material vestswith IMS Learning Resources Pvt.Ltd.
No part of this materials either in part oras a whole shall be copied,printed,electronically reproduced,sold or distributed without the written
consent of IMS Learing Resources Pvt.Ltd.and any such violation would entail initiation of suitable legal proceedings.

Copyright

Login ID:8FA03376/Student Name:ANSHULGARG/Overall Score:7

3)

Nil

4)

None of these

Explanation:
G and H are the only letters in the word FORESIGHT which have as many letters between them in the word. FORESIGHT as the number
of letters between them in the English alphabet. Hence, [1].

ite
d

Question: 81
Choose the correct alternative.

One

2)

Two

3)

Three

4)

Four

iv
at
e

1)

Li
m

How many such digits are there in the number '10743897', each of which changes its position by 1 place when the digits are arranged in
ascending order within the number?

rc

es

Question: 82
Find the odd man out from the series.

197

3)

290

4)

401

es

2)

120

ou

120, 197, 290, 401, 530


1)

Pr

Explanation:
Original number: 10743897
Ascending order: 01347789
The 4 digits shown in bold moved their positions by 1 place. Hence, [4].

ni

ng

Explanation:

1)

726

2)

4, 9, 29, 125, 726

Le

ar

Question: 83
Find the odd man out from the series.

IM

29

3)

125

4)

Explanation:

IMS Learning Resources Pvt.Ltd.,Mumbai.All copyrights to this material vestswith IMS Learning Resources Pvt.Ltd.
No part of this materials either in part oras a whole shall be copied,printed,electronically reproduced,sold or distributed without the written
consent of IMS Learing Resources Pvt.Ltd.and any such violation would entail initiation of suitable legal proceedings.

Copyright

Login ID:8FA03376/Student Name:ANSHULGARG/Overall Score:7

ite
d

Question: 84
Find the odd man out from the series.

120

2)

25

3)

504

4)

210

iv
at
e

1)

Li
m

25, 60, 120, 210, 504

ou

rc

es

Pr

Explanation:

es

Question: 85
Three out of four sentences contribute to one main idea. Choose the one that is not related to the others.
Soon motorists will have to show 'pollution under control' certificates at petrol pumps to refuel their vehicles, with the Delhi
government taking an in-principle decision to this effect primarily to check vehicular pollution in the city.

2)

Indicating that it may take couple of months to implement the decision, the Delhi government will first launch a campaign to
educate people and other stakeholders about the issue so that people voluntarily abide by the directives.

3)

According to the decision, motorists not having a PUC certificate will not be able to fill their tanks as it will be made
mandatory for petrol pumps to check the PUC before selling diesel and petrol.

4)

An expert committee last year had recommended a series of measures including significantly hiking parking fee and other
road taxes to encourage people to use public transport.

Le

ar

ni

ng

1)

IM

Explanation:
Sentences [1], [2] and [3] contribute to one main idea: the decision to mandate PUC certificates for the refuelling of vehicles. Sentence [4],
however, talks about measures to encourage the use of public transport. Hence, [4].
Question: 86
Three out of four sentences contribute to one main idea. Choose the one that is not related to the others.
1)

Although humans have established many types of societies throughout history, anthropologists tend to classify different
societies according to the degree to which different groups within a society have unequal access to advantages such as
resources, prestige or power.

IMS Learning Resources Pvt.Ltd.,Mumbai.All copyrights to this material vestswith IMS Learning Resources Pvt.Ltd.
No part of this materials either in part oras a whole shall be copied,printed,electronically reproduced,sold or distributed without the written
consent of IMS Learing Resources Pvt.Ltd.and any such violation would entail initiation of suitable legal proceedings.

Copyright

Login ID:8FA03376/Student Name:ANSHULGARG/Overall Score:7

3)

The word 'society' may also refer to an organized voluntary association of people for religious, benevolent, cultural,
scientific, political, patriotic or other purposes and may even refer to a social organism such as an ant colony or any
cooperative aggregate.

4)

In sociology, an industrial society refers to a society driven by the use of technology to enable mass production, supporting
a large population with a high capacity for division of labour.

ite
d

2)

In a pre-industrial society, food production, which is carried out through the use of human and animal labour, is the main
economic activity and these societies can be subdivided according to their level of technology and their method of
producingfood.

Li
m

Explanation:
All sentences except [3] talk about different types of societies, as classified by anthropologists. Sentence [3] discusses a completely
different interpretation of the word 'society'. Hence, [3].
Question: 87
Three out of four sentences contribute to one main idea. Choose the one that is not related to the others.

Yoga came to the attention of an educated western public in the mid-19th century along with other topics of Indian
philosophy.

2)

The first Hindu teacher to actively advocate and disseminate aspects of yoga to a western audience, Swami Vivekananda,
toured Europe and the United Statesin the 1890s.

3)

During the 1910s and 1920s in the USA, yoga suffered a period of bad publicity largely due to the backlash against
immigration, a rise in puritanical values, and a number of scandals.

4)

American educational institutions have always supported evidence-based public policy that encourages healthy lifestyles
and the safe enjoyment of sports andother physical activities.

es

Pr

iv
at
e

1)

ou

rc

Explanation:
Sentences [1], [2] and [3] are about the evolution of yoga in the west. Sentence [4] is about the ideology of American educational
institutions in general, and has nothing to do with yoga. Hence, [4].

es

Question: 88
Three out of four sentences contribute to one main idea. Choose the one that is not related to the others.
State-run fuel retailers have slashed the price of petrol by Rs. 1.51 a litre, excluding state taxes, on the back of a decline in
international oil prices and a firming up of the rupee exchange rate.

2)

Indian Oil Corporation, India's largest fuel retailer, said the downward trend in international prices has warranted a cut in the
price of non-subsidized cooking gas (LPG) by Rs. 19 per 14.2-kg cylinder.

3)

The retail price of petrol in South Africa will decrease by 67 cents to 13.66 rand a litre from next Wednesday, its lowest
since the start of the year, the department of energy said on Friday.

4)

The PRA said that the wholesale cost of refined petrol on the Rotterdam spot market had also dropped significantly in the
last month from 185 a tonne to 170 a tonne.

Le

ar

ni

ng

1)

IM

Explanation:
All sentences except [2] talk about a fall in petrol prices across countries. Sentence [2] is about the drop in the price of LPG/cooking gas.
Hence, [2].
Question: 89
Three out of four sentences contribute to one main idea. Choose the one that is not related to the others.
1)

These homologous traits and sequences are more similar among species that share a more recent common ancestor, and
can be used to reconstruct evolutionary histories, using both existing species and the fossil record.

IMS Learning Resources Pvt.Ltd.,Mumbai.All copyrights to this material vestswith IMS Learning Resources Pvt.Ltd.
No part of this materials either in part oras a whole shall be copied,printed,electronically reproduced,sold or distributed without the written
consent of IMS Learing Resources Pvt.Ltd.and any such violation would entail initiation of suitable legal proceedings.

Copyright

Login ID:8FA03376/Student Name:ANSHULGARG/Overall Score:7

2)

Evolutionary processes give rise to diversity at every level of biological organisation, including species, individual
organisms and molecules such as DNA and proteins.

3)

Repeated speciation and the divergence of life can be inferred from shared sets of biochemical and morphological traits, or
by shared DNA sequences.

4)

Biology is a natural science concerned with the study of life and living organisms,including their structure, function, growth,
evolution, distribution and taxonomy.

ite
d

Explanation:
Sentences [1], [2] and [3] discuss the evolutionary process. Sentence [4], on the other hand, is just a definition of biology. Hence, [4].

iv
at
e

Li
m

Question: 90
A statement that provides some information is followed by two other statements, numbered 1 and 2. Classify them on the basis of the
given options.
At their current rate of destruction, there will be no tropical rainforests left in the world within a century.
Statement 1: Citizens in tropical regions are planting new trees to take the place of the ones that are cut down.
Statement 2: The governments of many tropical countries are taking steps to curb the destruction of their rainforests.
Both statements weaken the given information.

2)

Both statements strengthen the given information.

3)

Both statements are neutral.

4)

Statement 1 weakens the given information and statement 2 strengthens it.

Pr

1)

rc

es

Explanation:
Both the statements weaken the information that tropical rainforests are declining irreversibly, by mentioning steps taken by the people of
these tropical countries and their governments, respectively, to reverse this trend. Hence, [1].

es

ou

Question: 91
A statement that provides some information is followed by two other statements, numbered 1 and 2. Classify them on the basis of the
given options.
The recent troubles involving the Malaysian Airlines planes have made people wary of air travel.
Statement 1: Air travel in general is in fact safer than road travel.
Statement 2: People have always been afraid of air travel the Malaysian Airlines incidents are just the latest excuse for the same.
Both statements weaken the given information.

2)

Both statements strengthen the given information.

3)

Statement 1 weakens the given information and statement 2 is neutral.

4)

Statement 1 is neutral and statement 2 weakens the given information.

ar

ni

ng

1)

IM

Le

Explanation:
The argument is that people have begun to think of air travel as unsafe due to particular incidents involving Malaysian Airlines planes.
Statement 1, which merely states a general fact about air travel, does not weaken this argument, as it does not address the cause-effect
connection between the Malaysian Airlines planes and people's fear of air travel. On the other hand, Statement 2 does weaken the
argument, as it states that fear of air travel predates these specific incidents. Hence, [4].
Question: 92
A statement that provides some information is followed by two other statements, numbered 1 and 2. Classify them on the basis of the
given options.
The Clean India campaign aims to clean up India by 2019.
Statement 1: If all Indians work together, the aims of the Clean India campaign may become a reality.
Statement 2: The campaign will be doomed to failure unless the Indian peoples mindset towards cleanliness is also changed.
1)

Statement 1 strengthens the given information and statement 2 is neutral.

IMS Learning Resources Pvt.Ltd.,Mumbai.All copyrights to this material vestswith IMS Learning Resources Pvt.Ltd.
No part of this materials either in part oras a whole shall be copied,printed,electronically reproduced,sold or distributed without the written
consent of IMS Learing Resources Pvt.Ltd.and any such violation would entail initiation of suitable legal proceedings.

Copyright

Login ID:8FA03376/Student Name:ANSHULGARG/Overall Score:7

2)

Statement 1 is neutral and statement 2 weakens the given information.

3)

Both statements are neutral.

4)

Statement 1 weakens the given information and statement 2 strengthens it.

Explanation:
The main statement is about the aim of the Clean India campaign. So, only statements that discuss the aims of the campaign can
strengthen or weaken it. The main statement is not about whether the aim is expected to be achieved or not. So, neither 1 nor 2 impacts it.
Hence, [3].

2)

[2]

3)

[3]

4)

[4]

Pr

[1]

es

1)

iv
at
e

Li
m

ite
d

Question: 93
Each question is followed by two statements, I and II.
Mark [1], if statement I and II together are not sufficient to answer the question.
Mark [2], if statement II alone is sufficient, but statement I alone is not sufficient to answer the question.
Mark [3], if both statements I and II together are sufficient, but neither of the statements alone is sufficient to answer the question.
Mark [4], if either statement alone is sufficient to answer the question.
Mohan walked 5 km towards the north, then turned right, walked a certain distance, then turned right again and walked a certain distance.
He then stopped. In which direction is Mohan now with respect to his starting point?
I. Mohan walked a total of 35 km.
II. Everytime Mohan turned, he walked twice the distance he walked previously.

IM

Le

ar

ni

ng

es

ou

rc

Explanation:

Question: 94
Each question is followed by two statements, I and II.
Mark [1], if statement I and II together are not sufficient to answer the question.
Mark [2], if statement II alone is sufficient, but statement I alone is not sufficient to answer the question.
Mark [3], if both statements I and II together are sufficient, but neither of the statements alone is sufficient to answer the question.
Mark [4], if either statement alone is sufficient to answer the question.

IMS Learning Resources Pvt.Ltd.,Mumbai.All copyrights to this material vestswith IMS Learning Resources Pvt.Ltd.
No part of this materials either in part oras a whole shall be copied,printed,electronically reproduced,sold or distributed without the written
consent of IMS Learing Resources Pvt.Ltd.and any such violation would entail initiation of suitable legal proceedings.

Copyright

Login ID:8FA03376/Student Name:ANSHULGARG/Overall Score:7

Amongst 6 friends A, B, C, D, E and F, who is the 3rd shortest? Given A is the tallest and B is the shortest.
I. C is taller than D and E
II. Amongst D and E, F is taller than one of them and shorter than the other.
1)

[1]

2)

[2]

3)

[3]

4)

[4]

iv
at
e

Li
m

ite
d

Explanation:
Using statement I alone we can only ascertain that C will be 2nd or 3rd tallest. So this statement alone is not enough to answer the
question.
Using statement II alone, we can only determine that D can be 2nd and 3rd tallest and E can be 4th or 5th tallest. So this statement alone
is not enough.
Combining both statements, we know that C is 2nd tallest, D and E will be 3rd and 5th tallest (not necessarily in that order) and F will be
4th tallest or 3rd shortest. Hence, [3].

[2]

3)

[3]

4)

[4]

ou

2)

es

[1]

1)

rc

es

Pr

Question: 95
Each question is followed by two statements, I and II.
Mark [1], if statement I and II together are not sufficient to answer the question.
Mark [2], if statement II alone is sufficient, but statement I alone is not sufficient to answer the question.
Mark [3], if both statements I and II together are sufficient, but neither of the statements alone is sufficient to answer the question.
Mark [4], if either statement alone is sufficient to answer the question.
How is B related to F?
I. A is the mother of B and C is the sister of D.
II. D is married to A and F is married to C.

IM

Le

ar

ni

ng

Explanation:

Question: 96
Each question is followed by two statements, I and II.
Mark [1], if statement I and II together are not sufficient to answer the question.
Mark [2], if statement II alone is sufficient, but statement I alone is not sufficient to answer the question.
Mark [3], if both statements I and II together are sufficient, but neither of the statements alone is sufficient to answer the question.

IMS Learning Resources Pvt.Ltd.,Mumbai.All copyrights to this material vestswith IMS Learning Resources Pvt.Ltd.
No part of this materials either in part oras a whole shall be copied,printed,electronically reproduced,sold or distributed without the written
consent of IMS Learing Resources Pvt.Ltd.and any such violation would entail initiation of suitable legal proceedings.

Copyright

Login ID:8FA03376/Student Name:ANSHULGARG/Overall Score:7

1)

[1]

2)

[2]

3)

[3]

4)

[4]

ite
d

Mark [4], if either statement alone is sufficient to answer the question.


In a class exam, Harshil is ranked 15th from bottom and Shirish is ranked 10th from top. How many students are in the class? (No two
students have the same rank)
I. Between Harshil and Shirish there are 4 students in ranks for the class exam.
II. Harshil is ranked higher than Shirish in the class exam.

iv
at
e

Li
m

Explanation:
Using statement I alone Harshil can either be ranked 5th from top or 15th from top, which means number of students can be either 19 or
29.
Using statement II alone, Harshil can be ranked anywhere between 1st to 9th from top. Combining both statements Harshil is ranked 5th
and number of students in class is 19.
Hence, [3].

[2]

3)

[3]

4)

[4]

es

2)

[1]

ng

1)

ou

rc

es

Pr

Question: 97
Each question is followed by two statements, I and II.
Mark [1], if statement I and II together are not sufficient to answer the question.
Mark [2], if statement II alone is sufficient, but statement I alone is not sufficient to answer the question.
Mark [3], if both statements I and II together are sufficient, but neither of the statements alone is sufficient to answer the question.
Mark [4], if either statement alone is sufficient to answer the question.
5 people A, B, C, D and E belong to one of the 5 communties i.e., Christian, Hindu, Muslim, Parsi or Sikh with no 2 people belonging to
the same community. A is a Christian and B is a Hindu. Is C is a Muslim?
I. Neither D nor E is a Muslim.
II. E is not a Parsi and D is a Sikh.

ar

ni

Explanation:
Using statement I alone as neither D nor E is a Muslim only C can be Muslim. So this statement alone is sufficient to answer the question.
Using statement II alone as D is a Sikh, E has to be Muslim (as he is not a Parsi) and C is a Parsi. So C is not a Muslim. This statement
alone can answer the question. Since either statement alone can answer the question. Hence, [4].

IM

Le

Question: 98
Refer to the data below and answer the questions that follow.

IMS Learning Resources Pvt.Ltd.,Mumbai.All copyrights to this material vestswith IMS Learning Resources Pvt.Ltd.
No part of this materials either in part oras a whole shall be copied,printed,electronically reproduced,sold or distributed without the written
consent of IMS Learing Resources Pvt.Ltd.and any such violation would entail initiation of suitable legal proceedings.

Copyright

Li
m

ite
d

Login ID:8FA03376/Student Name:ANSHULGARG/Overall Score:7

Charles

2)

Dorothy

3)

Either Charles or Dorothy

4)

Cannot be determined

Pr

1)

iv
at
e

Who among the following likes Banana?

es

ou

rc

es

Explanation:

ng

Hence, [1].

Marcus

2)

Stanley

3)

Romney

Cannot be determined

IM

4)

Le

1)

ar

ni

Question: 99
Refer to the data below and answer the questions that follow.
Who among the following likes Hazelnut?

Explanation:
Refer to the data below and answer the questions that follow. Hence, [4].
Question: 100
Refer to the data below and answer the questions that follow.
Which dry fruit does Marcus like?
1)

Hazelnut

IMS Learning Resources Pvt.Ltd.,Mumbai.All copyrights to this material vestswith IMS Learning Resources Pvt.Ltd.
No part of this materials either in part oras a whole shall be copied,printed,electronically reproduced,sold or distributed without the written
consent of IMS Learing Resources Pvt.Ltd.and any such violation would entail initiation of suitable legal proceedings.

Copyright

Login ID:8FA03376/Student Name:ANSHULGARG/Overall Score:7

2)

Apricot

3)

Walnut

4)

Cannot be determined

Li
m

ite
d

Explanation:

iv
at
e

Hence, [4].

2)

Dorothy

3)

Stanley

4)

Romney

es

Charles

rc

1)

Pr

Question: 101
Refer to the data below and answer the questions that follow.
Who among the following likes a fruit and dry fruit whose names begin with the same letter?

ar

ni

ng

es

ou

Explanation:

Le

Hence, [4].

IM

Question: 102
Refer to the data below and answer the questions that follow.
Who among the following like dry fruits that begin with the same letter?
1)

Stanley and Marcus

2)

Romney and Watson

3)

Charles and Watson

4)

Romney and Dorothy

Explanation:

IMS Learning Resources Pvt.Ltd.,Mumbai.All copyrights to this material vestswith IMS Learning Resources Pvt.Ltd.
No part of this materials either in part oras a whole shall be copied,printed,electronically reproduced,sold or distributed without the written
consent of IMS Learing Resources Pvt.Ltd.and any such violation would entail initiation of suitable legal proceedings.

Copyright

ite
d

Login ID:8FA03376/Student Name:ANSHULGARG/Overall Score:7

Li
m

Hence, [2].

Cashew

2)

Pistachio

3)

Hazelnut

4)

Cannot be determined

Pr

1)

iv
at
e

Question: 103
Refer to the data below and answer the questions that follow.
If the person who likes Mango also likes Walnut, what is the favourite dry fruit of the person who likes Pineapple?

ng

es

ou

rc

es

Explanation:

Hence, [3].

IM

Le

ar

ni

Question: 104
Refer to the data below and answer the questions that follow.

IMS Learning Resources Pvt.Ltd.,Mumbai.All copyrights to this material vestswith IMS Learning Resources Pvt.Ltd.
No part of this materials either in part oras a whole shall be copied,printed,electronically reproduced,sold or distributed without the written
consent of IMS Learing Resources Pvt.Ltd.and any such violation would entail initiation of suitable legal proceedings.

Copyright

iv
at
e

Li
m

ite
d

Login ID:8FA03376/Student Name:ANSHULGARG/Overall Score:7

2)

15%

3)

25%

4)

30%

es

10%

rc

1)

Pr

What is the approximate maximum annual percent change in exchange rate of any currency?

IM

Le

ar

ni

ng

es

ou

Explanation:

IMS Learning Resources Pvt.Ltd.,Mumbai.All copyrights to this material vestswith IMS Learning Resources Pvt.Ltd.
No part of this materials either in part oras a whole shall be copied,printed,electronically reproduced,sold or distributed without the written
consent of IMS Learing Resources Pvt.Ltd.and any such violation would entail initiation of suitable legal proceedings.

Copyright

rc

ou

Question: 105
Refer to the data below and answer the questions that follow.

es

Pr

iv
at
e

Li
m

ite
d

Login ID:8FA03376/Student Name:ANSHULGARG/Overall Score:7

Which of the following statements is incorrect?

No currency exhibited increase in exchange rate every year with respect to previous year.

2)

There is at least one year in which exchange rates of all the currencies increased with respect to previous year.

3)

Exchange rates of US Dollar and Euro always changed in tandem i.e. exchange rates of both the currencies increased or
decreased together with respect to their respective values previous year.

4)

There is exactly one year in which exchange rates of all the currencies decreased with respect to previous year.

ng

es

1)

Le

ar

ni

Explanation:
From visual inspection, it can be seen that there has been no year in which the exchange rates of all the currencies decreased with
respect to their values at the end of previous year. Therefore statement [4] is incorrect. All other statements are correct. Hence, [4].

Question: 106
Refer to the data below and answer the questions that follow.

IM

If Anand bought one unit of all the five currencies (i.e. US Dollar, Australian Dollar, British Pound, Euro and Singapore Dollar) on 31st
December 2007 and sold them all on 31stDecember 2013, what was his approximate percent profit?
1)

50%

2)

40%

3)

60%

4)

30%

Explanation:

IMS Learning Resources Pvt.Ltd.,Mumbai.All copyrights to this material vestswith IMS Learning Resources Pvt.Ltd.
No part of this materials either in part oras a whole shall be copied,printed,electronically reproduced,sold or distributed without the written
consent of IMS Learing Resources Pvt.Ltd.and any such violation would entail initiation of suitable legal proceedings.

Copyright

Li
m

ite
d

Login ID:8FA03376/Student Name:ANSHULGARG/Overall Score:7

iv
at
e

Question: 107
Refer to the data below and answer the questions that follow.

2)

Australian Dollar

3)

British Pound

4)

Singapore Dollar

es

US Dollar

rc

1)

Pr

The percentage increase in exchange rate between 31stDecember 2007 and 31stDecember 2013 of which of the following currencies was
highest?

IM

Le

ar

ni

ng

es

ou

Explanation:

Question: 108
Refer to the data below and answer the questions that follow.
What is the average exchange rate of British Pound over the given period?

IMS Learning Resources Pvt.Ltd.,Mumbai.All copyrights to this material vestswith IMS Learning Resources Pvt.Ltd.
No part of this materials either in part oras a whole shall be copied,printed,electronically reproduced,sold or distributed without the written
consent of IMS Learing Resources Pvt.Ltd.and any such violation would entail initiation of suitable legal proceedings.

Copyright

Login ID:8FA03376/Student Name:ANSHULGARG/Overall Score:7

1)

75

2)

80

3)

85

4)

87.5

Pr

iv
at
e

Li
m

ite
d

Explanation:

es

Question: 109
Choose the correct alternative.

ou

rc

The angle of elevation of the top of a tower from a point is 45 degrees. If the distance of the point from the base of the tower is 100 m,
what is the height of the tower?

50 m

3)

100 m

2)

es

1)

ng

4)

IM

Le

ar

ni

Explanation:

IMS Learning Resources Pvt.Ltd.,Mumbai.All copyrights to this material vestswith IMS Learning Resources Pvt.Ltd.
No part of this materials either in part oras a whole shall be copied,printed,electronically reproduced,sold or distributed without the written
consent of IMS Learing Resources Pvt.Ltd.and any such violation would entail initiation of suitable legal proceedings.

Copyright

Li
m

ite
d

Login ID:8FA03376/Student Name:ANSHULGARG/Overall Score:7

iv
at
e

Question: 110
Choose the correct alternative.

2)

3 : 20

3)

3 : 40

4)

1 : 40

es

1:5

rc

1)

Pr

The roots of the equation ax2+ bx + c = 0 are 8 and 5. Find the ratio b : c.

ni

Question: 111
Choose the correct alternative.

ng

es

ou

Explanation:

2)

Rs. 135

3)

Rs. 124

4)

ar

Rs. 120

Le

1)

A shopkeeper bought an article for Rs. 1200 and sold it at a profit of 35%. If he wants to increase the profit percentage to 45%, by what
value should he increase the selling price of the article?

IM

Rs. 144

Explanation:
An increase from 35% to 45% in profit means that there is an increase of 10% profit on Rs. 1200 = 0.1 1200 = Rs. 120. Hence, [1].
Question: 112
Choose the correct alternative.
What is the minimum value of x2+ 6x + 15?

IMS Learning Resources Pvt.Ltd.,Mumbai.All copyrights to this material vestswith IMS Learning Resources Pvt.Ltd.
No part of this materials either in part oras a whole shall be copied,printed,electronically reproduced,sold or distributed without the written
consent of IMS Learing Resources Pvt.Ltd.and any such violation would entail initiation of suitable legal proceedings.

Copyright

Login ID:8FA03376/Student Name:ANSHULGARG/Overall Score:7

1)

2)

3)

15

4)

12

ite
d

Explanation:

Li
m

Question: 113
Choose the correct alternative.

(3, 2)

2)

(5, 8)

3)

(1, 3)

4)

(1, 6)

Pr

1)

iv
at
e

836597P435Q is an integer which is divisible by 18. P and Q denote different non-zero single digit numbers. From the given options, what
can represent values of P and Q respectively?

es

ou

rc

es

Explanation:

Question: 114
Choose the correct alternative.

30

3)

120

4)

24

IM

Explanation:

ni

2)

ar

60

Le

1)

ng

In how many ways can you rearrange the letters of the word MESSAGE such that the first and the last letters are the same?

Question: 115
Choose the correct alternative.
Three years ago, ratio of the ages of Anil and his elder brother, Sunil was 2 : 5. Five years from now, the ratio of ages of Sunil and Anil will
be 7 : 4. What is the present age of Sunil?

IMS Learning Resources Pvt.Ltd.,Mumbai.All copyrights to this material vestswith IMS Learning Resources Pvt.Ltd.
No part of this materials either in part oras a whole shall be copied,printed,electronically reproduced,sold or distributed without the written
consent of IMS Learing Resources Pvt.Ltd.and any such violation would entail initiation of suitable legal proceedings.

Copyright

Login ID:8FA03376/Student Name:ANSHULGARG/Overall Score:7

1)

21 years

2)

23 years

3)

14 years

4)

17 years

iv
at
e

Li
m

ite
d

Explanation:

Pr

Question: 116
Choose the correct alternative.

es

The sum of the first, third, fifth, seventh and ninth terms of an A.P is 135. What is the fifth term of the AP?
30

2)

27

3)

21

4)

Cannot be determined

es

ou

rc

1)

ar

ni

ng

Explanation:

IM

Le

Question: 117
Choose the correct alternative.

IMS Learning Resources Pvt.Ltd.,Mumbai.All copyrights to this material vestswith IMS Learning Resources Pvt.Ltd.
No part of this materials either in part oras a whole shall be copied,printed,electronically reproduced,sold or distributed without the written
consent of IMS Learing Resources Pvt.Ltd.and any such violation would entail initiation of suitable legal proceedings.

Copyright

ite
d

Login ID:8FA03376/Student Name:ANSHULGARG/Overall Score:7

Li
m

1)

iv
at
e

2)

3)

Pr

4)

IM

Le

ar

ni

ng

es

ou

rc

es

Explanation:

Question: 118
Choose the correct alternative.
It takes 3 hours 40 minutes to go from city A to city B by train and 4 hours 10 minutes by taxi. What is the ratio of the speed of taxi to the
speed of train?

IMS Learning Resources Pvt.Ltd.,Mumbai.All copyrights to this material vestswith IMS Learning Resources Pvt.Ltd.
No part of this materials either in part oras a whole shall be copied,printed,electronically reproduced,sold or distributed without the written
consent of IMS Learing Resources Pvt.Ltd.and any such violation would entail initiation of suitable legal proceedings.

Copyright

Login ID:8FA03376/Student Name:ANSHULGARG/Overall Score:7

1)

3:4

2)

11:25

3)

6:25

4)

22:25

Li
m

ite
d

Explanation:

iv
at
e

Question: 119
Choose the correct alternative.

Harish bought a share of a company by paying Rs. 90. The face value of the share is Rs. 100. The share paid a dividend of 7%. After
receiving the dividend, he sold the share for Rs. 74. What was the percent loss he suffered in the transaction?
25.56%

2)

10%

3)

17.77%

4)

None of the above

es

Pr

1)

2)

IM

3)

Le

1)

ar

ni

Question: 120
Choose the correct alternative.

ng

es

ou

rc

Explanation:

4)

Explanation:

IMS Learning Resources Pvt.Ltd.,Mumbai.All copyrights to this material vestswith IMS Learning Resources Pvt.Ltd.
No part of this materials either in part oras a whole shall be copied,printed,electronically reproduced,sold or distributed without the written
consent of IMS Learing Resources Pvt.Ltd.and any such violation would entail initiation of suitable legal proceedings.

Copyright

Login ID:8FA03376/Student Name:ANSHULGARG/Overall Score:7

ite
d

Question: 121
Choose the correct alternative.

0.45

2)

0.40

3)

0.50

4)

0.60

iv
at
e

1)

Li
m

100 cups numbered from 1 to 100 are kept on a table. Ramesh hits a particular cup with a ball. What is the probability that the number on
that ball is a multiple of 4 or 5?

rc

es

Pr

Explanation:

es

ou

Question: 122
Choose the correct alternative.

2)

496.75 sq.cm

3)

552.75 sq.cm

4)

484.25 sq.cm

IM

Le

ar

Explanation:

ng

569.25 sq.cm

ni

1)

A rectangular sheet of paper of length 33 cm and breadth 12 is rolled tomake aright circular cylinder of height 12 cm. Find the total surface
area of the cylinder? (Consider that both ends of the cylinder are made from a sheet other than the given one).

Question: 123
Choose the correct alternative.

IMS Learning Resources Pvt.Ltd.,Mumbai.All copyrights to this material vestswith IMS Learning Resources Pvt.Ltd.
No part of this materials either in part oras a whole shall be copied,printed,electronically reproduced,sold or distributed without the written
consent of IMS Learing Resources Pvt.Ltd.and any such violation would entail initiation of suitable legal proceedings.

Copyright

Login ID:8FA03376/Student Name:ANSHULGARG/Overall Score:7

What should be percent dividend paid by a share so that a share purchased for Rs. 80 and sold for Rs. 64 will result in no-profit no-loss
situation? Assume that the face value of the share is Rs. 100.
1)

20%

2)

25%

3)

16%

4)

36%

Li
m

ite
d

Explanation:

105

2)

110

Pr

1)

iv
at
e

Question: 124
Choose the correct alternative.

3)

es

4)

ng
ni

Question: 125
Choose the correct alternative.

es

ou

rc

Explanation:

ar

A bag contains 12 black balls, 23 white balls and 15 red balls. A boy takes out a ball from the bag which turns out to be red. He then takes
out another ball. What is the probability that the second ball drawn from the bag is also red?

Le

1)

IM

3)

2)

4)
Explanation:

IMS Learning Resources Pvt.Ltd.,Mumbai.All copyrights to this material vestswith IMS Learning Resources Pvt.Ltd.
No part of this materials either in part oras a whole shall be copied,printed,electronically reproduced,sold or distributed without the written
consent of IMS Learing Resources Pvt.Ltd.and any such violation would entail initiation of suitable legal proceedings.

Copyright

Login ID:8FA03376/Student Name:ANSHULGARG/Overall Score:7

ite
d

Question: 126
Choose the correct alternative.

Rs.56,000

2)

Rs.28,000

3)

Rs.65,000

4)

None of these

iv
at
e

1)

Li
m

A, B and C invest an amount in a partnership firm. A invests Rs.28,000; B invests Rs.18,000 and C invests Rs.42,000. B and C back out
of the business 6 months and 4 months respectively before completion of the year. Find the share of C in the profit of Rs.1,30000.

rc

es

Pr

Explanation:

1)

36 units

2)

48 units

3)

Le

ar

ni

ng

es

ou

Question: 127
Choose the correct alternative.

IM

20 units

4)

24 units

Explanation:

IMS Learning Resources Pvt.Ltd.,Mumbai.All copyrights to this material vestswith IMS Learning Resources Pvt.Ltd.
No part of this materials either in part oras a whole shall be copied,printed,electronically reproduced,sold or distributed without the written
consent of IMS Learing Resources Pvt.Ltd.and any such violation would entail initiation of suitable legal proceedings.

Copyright

Li
m

ite
d

Login ID:8FA03376/Student Name:ANSHULGARG/Overall Score:7

0.01534

2)

0.1534

3)

1.534

4)

15.34

rc

es

1)

Pr

iv
at
e

Question: 128
Choose the correct alternative.

IM

Le

ar

ni

ng

es

ou

Explanation:

Question: 129
Choose the correct alternative.
Ram Bahadur invested a sum of Rs. 5000 on simple interest at 10% per annum for 3 years. If he had invested the same amount on
compound interest at the same rate and for the same period of time, how much extra interest would he have received?(Assume interest is
compounded annually)
1)

Rs. 115

IMS Learning Resources Pvt.Ltd.,Mumbai.All copyrights to this material vestswith IMS Learning Resources Pvt.Ltd.
No part of this materials either in part oras a whole shall be copied,printed,electronically reproduced,sold or distributed without the written
consent of IMS Learing Resources Pvt.Ltd.and any such violation would entail initiation of suitable legal proceedings.

Copyright

Login ID:8FA03376/Student Name:ANSHULGARG/Overall Score:7

2)

Rs. 151

3)

Rs. 155

4)

Rs. 5155

ite
d

Explanation:

Li
m

Question: 130
Choose the correct alternative.

Rs.32340

2)

Rs.33340

3)

Rs.23320

4)

Rs.34320

Pr

1)

iv
at
e

A circular jogging track, 7m wide has to be constructed around a circular ground of radius, 70 m. If the cost of construction is Rs.10 per
square meter, then find the total cost incurred.

es

ou

rc

es

Explanation:

ng

Question: 131
Choose the correct alternative.

2)

3)

12

4)

16

IM

Explanation:

ar

Le

1)

ni

Three numbers are in a ratio 2 : 3 : 5 and their LCM is 360. What is their HCF?

Question: 132

IMS Learning Resources Pvt.Ltd.,Mumbai.All copyrights to this material vestswith IMS Learning Resources Pvt.Ltd.
No part of this materials either in part oras a whole shall be copied,printed,electronically reproduced,sold or distributed without the written
consent of IMS Learing Resources Pvt.Ltd.and any such violation would entail initiation of suitable legal proceedings.

Copyright

Login ID:8FA03376/Student Name:ANSHULGARG/Overall Score:7

Choose the correct alternative.

1)

30 and 45

2)

30 and 90

3)

45 and 90

4)

15 and 45

Explanation:
Among the given pairs, only 15 and 45 are the numbers whose HCF (15) and LCM (45) differ by 30. Hence, [4].

Li
m

Question: 133
Choose the correct alternative.

ite
d

The difference between the HCF and LCM of two numbers is 30. Which of the following can be the two numbers?

It is divisible by 11

2)

It is divisible by 6

3)

It is divisible by 9

4)

It is divisible by 7

Pr

1)

iv
at
e

Which of the following is true about the absolute value of the difference between a two digit number and the number obtained by reversing
its digits?

es

ou

rc

es

Explanation:

Question: 134
Choose the correct alternative.

ng

Average age of 20 people in a group is 47 years. One person aged 77 years joins the group while another person aged 37 years leaves
the group. What will be the new average age of the group?
47 years

2)

49 years

3)

45 years

4)

None of these

Le

ar

ni

1)

IM

Explanation:

Question: 135
Choose the correct alternative.
The average of the sum and difference of two positive numbers is 16. If one of the numbers is 9, find the other number.
1)

IMS Learning Resources Pvt.Ltd.,Mumbai.All copyrights to this material vestswith IMS Learning Resources Pvt.Ltd.
No part of this materials either in part oras a whole shall be copied,printed,electronically reproduced,sold or distributed without the written
consent of IMS Learing Resources Pvt.Ltd.and any such violation would entail initiation of suitable legal proceedings.

Copyright

Login ID:8FA03376/Student Name:ANSHULGARG/Overall Score:7

2)

16

3)

12

4)

25

ite
d

Explanation:

Li
m

Question: 136
Choose the correct alternative.

13%

2)

16%

3)

14%

4)

12%

Pr

1)

iv
at
e

The difference between simple interest and compound interest (compounded annually) at the same rate of interest on Rs.5000 for 2 years
is Rs.72. The rate of interest is:

IM

Le

ar

ni

ng

es

ou

rc

es

Explanation:

Question: 137
Choose the correct alternative.

1)

18

IMS Learning Resources Pvt.Ltd.,Mumbai.All copyrights to this material vestswith IMS Learning Resources Pvt.Ltd.
No part of this materials either in part oras a whole shall be copied,printed,electronically reproduced,sold or distributed without the written
consent of IMS Learing Resources Pvt.Ltd.and any such violation would entail initiation of suitable legal proceedings.

Copyright

Login ID:8FA03376/Student Name:ANSHULGARG/Overall Score:7

2)

3)

4)

10

ite
d

Explanation:

Li
m

Question: 138
Choose the correct alternative.

9.09%

2)

14.28%

3)

25%

4)

None of these

Pr

1)

iv
at
e

A container consists of salt and water in the ratio 1 : 4. What percentage of the solution should be replaced with salt so that the
concentration of salt in mixture is 40%?

ar

ni

ng

es

ou

rc

es

Explanation:

Le

Question: 139
Choose the correct alternative.

IM

Bottles A and B contain a mixture of petrol and kerosene. The ratio of petrol to kerosene in bottle A is 4 : 7 and the ratio of petrol to
kerosene in bottle B is 2 : 1. 550 ml of the mixture from bottle A and 225 ml of the mixture from bottle B are mixed together to form a new
mixture. What is the percentage of kerosene in this mixture?
1)

33.5%

2)

46.9%

3)

49.7%

4)

54.8%

Explanation:

IMS Learning Resources Pvt.Ltd.,Mumbai.All copyrights to this material vestswith IMS Learning Resources Pvt.Ltd.
No part of this materials either in part oras a whole shall be copied,printed,electronically reproduced,sold or distributed without the written
consent of IMS Learing Resources Pvt.Ltd.and any such violation would entail initiation of suitable legal proceedings.

Copyright

ite
d

Login ID:8FA03376/Student Name:ANSHULGARG/Overall Score:7

Question: 140
Given below are 2 quantities A and B. You are to compare the 2 quantities and
B

Li
m

Mark [1] if A

2)

[2]

3)

[3]

4)

[4]

Pr

[1]

es

1)

iv
at
e

Mark [2] if A
B
Mark [3] if A = B
Mark [4] if the relationship between A and B cannot be determined.
Quantity A:
Largest distance between 2 points of a cuboid of dimensions 5 m, 12 m and 13 m
Quantity B: 20 m

es

ou

rc

Explanation:

Mark [1] if A

ng

Question: 141
Given below are 2 quantities A and B. You are to compare the 2 quantities and
B

Le

ar

ni

Mark [2] if A
B
Mark [3] if A = B
Mark [4] if the relationship between A and B cannot be determined.
Quantity A: Roots of the equation x2 5x + 6 = 0
Quantity B: Roots of equation x2 9x + 20 = 0

[1]

IM

1)
2)

[2]

3)

[3]

4)

[4]

Explanation:

IMS Learning Resources Pvt.Ltd.,Mumbai.All copyrights to this material vestswith IMS Learning Resources Pvt.Ltd.
No part of this materials either in part oras a whole shall be copied,printed,electronically reproduced,sold or distributed without the written
consent of IMS Learing Resources Pvt.Ltd.and any such violation would entail initiation of suitable legal proceedings.

Copyright

Login ID:8FA03376/Student Name:ANSHULGARG/Overall Score:7

Question: 142
Given below are 2 quantities A and B. You are to compare the 2 quantities and
B

ite
d

Mark [1] if A

2)

[2]

3)

[3]

4)

[4]

iv
at
e

[1]

Pr

1)

Li
m

Mark [2] if A
B
Mark [3] if A = B
Mark [4] if the relationship between A and B cannot be determined.
Quantity A: Probability of getting a sum of 7 when 2 six-sided unbiased die are rolled
Quantity B: Probability of getting a sum of 6 when 2 six-sided unbiased die are rolled

es

ou

rc

es

Explanation:

ni

Mark [1] if A

ng

Question: 143
Given below are 2 quantities A and B. You are to compare the 2 quantities and

IM

Le

ar

Mark [2] if A
B
Mark [3] if A = B
Mark [4] if the relationship between A and B cannot be determined.

1)

[1]

2)

[2]

3)

[3]

IMS Learning Resources Pvt.Ltd.,Mumbai.All copyrights to this material vestswith IMS Learning Resources Pvt.Ltd.
No part of this materials either in part oras a whole shall be copied,printed,electronically reproduced,sold or distributed without the written
consent of IMS Learing Resources Pvt.Ltd.and any such violation would entail initiation of suitable legal proceedings.

Copyright

Login ID:8FA03376/Student Name:ANSHULGARG/Overall Score:7

4)

[4]

Explanation:

1)

[1]

2)

[2]

3)

[3]

4)

[4]

Pr

Mark [2] if A
B
Mark [3] if A = B
Mark [4] if the relationship between A and B cannot be determined.
Quantity A: Sum of all the terms of the arithmetic progression 1, 4, 7, ......., 52
Quantity B: Sum of all the terms of the infinite geometric progression 240, 120, 60..........

Li
m

iv
at
e

Mark [1] if A

ite
d

Question: 144
Given below are 2 quantities A and B. You are to compare the 2 quantities and

ng

es

ou

rc

es

Explanation:

Mark [1] if A

ni

Question: 145
Given below are 2 quantities A and B. You are to compare the 2 quantities and
B

Le

ar

Mark [2] if A
B
Mark [3] if A = B
Mark [4] if the relationship between A and B cannot be determined.

IM

Given 2 equations
5a + 16b = 84
7a + 22b = 116
Quantity A: value of 'a'
Quantity B: value of 'b'
1)

[1]

2)

[2]

3)

[3]

IMS Learning Resources Pvt.Ltd.,Mumbai.All copyrights to this material vestswith IMS Learning Resources Pvt.Ltd.
No part of this materials either in part oras a whole shall be copied,printed,electronically reproduced,sold or distributed without the written
consent of IMS Learing Resources Pvt.Ltd.and any such violation would entail initiation of suitable legal proceedings.

Copyright

Login ID:8FA03376/Student Name:ANSHULGARG/Overall Score:7

4)

[4]

Explanation:

ite
d

Question: 146
Choose the correct alternative.

24

2)

42

3)

32

4)

38

iv
at
e

1)

Li
m

The HCF, LCM and ratio of two natural numbers are 6, 72 and 3 : 4 respectively. What is the sum of the two numbers?

ou

rc

es

Pr

Explanation:

IM

Le

ar

ni

ng

es

Question: 147
Refer to the pie-chart below and answer the questions that follow.

Students from Economics background from District-1 who won the scholarship are what percent of the total number of students who won
the scholarship from the four districts combined?

IMS Learning Resources Pvt.Ltd.,Mumbai.All copyrights to this material vestswith IMS Learning Resources Pvt.Ltd.
No part of this materials either in part oras a whole shall be copied,printed,electronically reproduced,sold or distributed without the written
consent of IMS Learing Resources Pvt.Ltd.and any such violation would entail initiation of suitable legal proceedings.

Copyright

Login ID:8FA03376/Student Name:ANSHULGARG/Overall Score:7

1)

14%

2)

24%

3)

21%

4)

Cannot be determined

ite
d

Explanation:
Total 40% students won the scholarship from District-1, out of which 35% are from Economics background. Therefore students from
Economics background from District-1 who won the scholarship are 40% 35% = 14% of total number of students who won the
scholarship from the four districts combined. Hence [1].

1)

120

2)

270

3)

192

4)

Cannot be determined

iv
at
e

Li
m

Question: 148
Refer to the pie-chart below and answer the questions that follow.
If 400 students from district-2 won the scholarship, how many students from Science background from District-1 won the scholarship?

ou

rc

es

Pr

Explanation:

es

Question: 149
Refer to the pie-chart below and answer the questions that follow.
What percent of students from Engineering background in the four districts have won the scholarship?
25%

2)

30%

3)

35%

4)

Cannot be determined

ni

ng

1)

Le

ar

Explanation:
Second pie chart contains information only of the break-up of background of students from
District-1. However we do not have information about other districts. Therefore this
question cannot be answered. Hence, [4].

IM

Question: 150
Refer to the pie-chart below and answer the questions that follow.
Percentage of students who won the scholarship in the four districts, who are not from Engineering background from District-1 is:
1)

10%

2)

14%

3)

90%

4)

86%

IMS Learning Resources Pvt.Ltd.,Mumbai.All copyrights to this material vestswith IMS Learning Resources Pvt.Ltd.
No part of this materials either in part oras a whole shall be copied,printed,electronically reproduced,sold or distributed without the written
consent of IMS Learing Resources Pvt.Ltd.and any such violation would entail initiation of suitable legal proceedings.

Copyright

Login ID:8FA03376/Student Name:ANSHULGARG/Overall Score:7

Explanation:

2)

1800

3)

2700

4)

3120

Li
m

2250

iv
at
e

1)

ite
d

Question: 151
Refer to the pie-chart below and answer the questions that follow.
If 315 students from Economics background from District-1 won the scholarship, what is the total number of students who won the
scholarship in the four districts combined?

IM

Le

ar

ni

ng

es

ou

Question: 152
Refer to the data below and answer the questions that follow.

rc

es

Pr

Explanation:

1)

140

2)

150

3)

155

4)

160

IMS Learning Resources Pvt.Ltd.,Mumbai.All copyrights to this material vestswith IMS Learning Resources Pvt.Ltd.
No part of this materials either in part oras a whole shall be copied,printed,electronically reproduced,sold or distributed without the written
consent of IMS Learing Resources Pvt.Ltd.and any such violation would entail initiation of suitable legal proceedings.

Copyright

Login ID:8FA03376/Student Name:ANSHULGARG/Overall Score:7

Explanation:

21

2)

42

3)

63

4)

70

Li
m

1)

ite
d

Question: 153
Refer to the data below and answer the questions that follow.

2)

105

3)

35

4)

37.5

rc

es

110

1)

ou

Question: 154
Refer to the data below and answer the questions that follow.

es

Pr

iv
at
e

Explanation:

Le

ar

ni

ng

Explanation:

IM

Question: 155
Refer to the data below and answer the questions that follow.

IMS Learning Resources Pvt.Ltd.,Mumbai.All copyrights to this material vestswith IMS Learning Resources Pvt.Ltd.
No part of this materials either in part oras a whole shall be copied,printed,electronically reproduced,sold or distributed without the written
consent of IMS Learing Resources Pvt.Ltd.and any such violation would entail initiation of suitable legal proceedings.

Copyright

Li
m

ite
d

Login ID:8FA03376/Student Name:ANSHULGARG/Overall Score:7

1)

4.6 cm

2)

4.8 cm

3)

3.7 cm

4)

2.3 cm

iv
at
e

Find the length of BM.

ou

rc

es

Pr

Explanation:

3.2 cm2

3)

4.4 cm2

4)

5.2 cm2

IM

Le

Explanation:

ng

2)

ni

2.2 cm2

ar

1)

es

Question: 156
Refer to the data below and answer the questions that follow.

Question: 157
Mark the word that does not mean the same as the capitalized word.
PEDANTIC
1)

Fastidious

IMS Learning Resources Pvt.Ltd.,Mumbai.All copyrights to this material vestswith IMS Learning Resources Pvt.Ltd.
No part of this materials either in part oras a whole shall be copied,printed,electronically reproduced,sold or distributed without the written
consent of IMS Learing Resources Pvt.Ltd.and any such violation would entail initiation of suitable legal proceedings.

Copyright

Login ID:8FA03376/Student Name:ANSHULGARG/Overall Score:7

2)

Superfluous

3)

Scrupulous

4)

Meticulous

Explanation:
'Pedantic' means 'over-concerned with formal rules and details'. Options [1], [3] and [4] are all synonyms, while 'superfluous' means 'extra'
or 'unnecessary'. Hence, [2].

ite
d

Question: 158
Mark the word that does not mean the same as the capitalized word.

Improvident

2)

Immoderate

3)

Extravagant

4)

Impetuous

iv
at
e

1)

Li
m

PRODIGAL

Pr

Explanation:
'Prodigal' means 'wastefully or recklessly extravagant'. Options [1], [2] and [3] are similar in meaning to the given word. 'Impetuous' means
'hasty or impulsive' and therefore is the correct choice. Hence, [4].

Bigoted

3)

Obviate

4)

Forthright

ou

2)

es

Deceptive

1)

rc

FALLACIOUS

es

Question: 159
Mark the word that means the same as the capitalized word.

ni

ng

Explanation:
'Fallacious' means 'deceptive' or 'logically unsound'. 'Bigoted' means 'utterly intolerant of any creed, belief or opinion that differs from one's
own'. 'Obviate' means 'anticipate and prevent or eliminate (difficulties, disadvantages, etc.) by effective measures'. 'Forthright' means
'frank or direct'. Hence, [1].

1)

Feeble

2)

INDOMITABLE

Le

ar

Question: 160
Mark the word that means the same as the capitalized word.

IM

Unconquerable

3)

Profane

4)

Overworked

Explanation:
The meaning of 'indomitable' is 'impossible to defeat'. Option [2] means the same as this. Hence, [2].
Question: 161
Mark the word that means the same as the capitalized word.

IMS Learning Resources Pvt.Ltd.,Mumbai.All copyrights to this material vestswith IMS Learning Resources Pvt.Ltd.
No part of this materials either in part oras a whole shall be copied,printed,electronically reproduced,sold or distributed without the written
consent of IMS Learing Resources Pvt.Ltd.and any such violation would entail initiation of suitable legal proceedings.

Copyright

Login ID:8FA03376/Student Name:ANSHULGARG/Overall Score:7

From the options below, mark the word which can best substitute 'overly critical and complaining'.
1)

Imperceptive

2)

Ostensible

3)

Affable

4)

Captious

Li
m

Question: 162
Four words are highlighted in each sentence. Identify the one that is inappropriately/incorrectly used.

ite
d

Explanation:
'Captious' is used to describe a person who is apt to notice and make much of trivial faults or defects. 'Ostensible' means 'apparent or
professed'. 'Affable' means 'cordial or friendly'. Hence, [4].

Incumbent

2)

Accepted

3)

Predictable

4)

Opposition

Pr

1)

iv
at
e

The incumbent party has accepted that it has been trounced by the opposition in the national election, but predictable, the party
offered cover to the frontman, claiming that the blame is 'collective'.

rc

es

Explanation:
'Predictable' is an adjective. 'Predictably' (the adverbial form) should be used instead, as the word is describing the action of offering
cover. Hence, [3].

ou

Question: 163
Four words are highlighted in each sentence. Identify the one that is inappropriately/incorrectly used.

Surveyed

3)

Alliance

4)

Live

ng

2)

ni

Travellers

ar

1)

es

The recent Frequent Business Traveller survey found that travellers are more loyal to their favourite airlines than to hotel brands: over
93% of those surveyed said they prefer to fly with a particular airline or alliance, while only 72% said they prefer to live at a particular
brand of hotels.

Le

Explanation:
'Live' is used when the sentence implies a long-term, indefinite or a permanent stay at a place. In the context of the above sentence, 'stay'
would be a more appropriate word, since the sentence talks of putting up at hotels, which is a short-term activity. Hence, [4].

IM

Question: 164
Four words are highlighted in each sentence. Identify the one that is inappropriately/incorrectly used.
International trade is, in principal, not different from domestic trade as the motivation and the behaviour of parties involved in a trade do
not change fundamentally regardless of whether trade is across a border or not.
1)

Principal

2)

Behaviour

3)

Fundamentally

IMS Learning Resources Pvt.Ltd.,Mumbai.All copyrights to this material vestswith IMS Learning Resources Pvt.Ltd.
No part of this materials either in part oras a whole shall be copied,printed,electronically reproduced,sold or distributed without the written
consent of IMS Learing Resources Pvt.Ltd.and any such violation would entail initiation of suitable legal proceedings.

Copyright

Login ID:8FA03376/Student Name:ANSHULGARG/Overall Score:7

4)

Whether

Explanation:
'Principal' means 'primary or chief' and can be used as a noun (most commonly to denote the head of an institution/department) or an
adjective. 'Principle', on the other hand, is a rule or law. 'In principle' means 'in essence'. So, 'principal' should be replaced by 'principle'.
Hence, [1].

ite
d

Question: 165
Four words are highlighted in each sentence. Identify the one that is inappropriately/incorrectly used.

Despite

2)

Dominate

3)

Increased

4)

Storage

iv
at
e

1)

Li
m

Despite the oceans' dominate role in energy storage, the term 'global warming' is also used to refer to increases in the average
temperature of the air and sea at the Earth's surface.

Explanation:
'Dominate' is a verb. In the context of the sentence, 'dominant' (an adjective describing 'role') would be the correct word. Hence, [2].

Pr

Question: 166
Four words are highlighted in each sentence. Identify the one that is inappropriately/incorrectly used.

Decreased

3)

Between

4)

According

rc

2)

ou

Incidents

es

1)

es

The incidents of child labour in the world decreased from 25% to 10% between 1960 and 2003, according to the World Bank.

ng

Explanation:
The correct usage in the context of the above sentence is 'incidence' and not 'incidents'. 'Incidents' are distinct events. 'Incidence' refers to
the rate of occurrence or influence of something. Hence, [1].

ni

Question: 167
The sentence below has a highlighted phrase. Replace the phrase with the correct option from the choices given below.

ar

As per the guidelines, any administrative breach committed inside the establishment is to be settled according to the provisions of the
Labour Law, taking into consideration exceptional circumstances the country has gone through.
according to

2)

in accordance to

3)

in accordance with

with accordance to

IM

4)

Le

1)

Explanation:
Options [1] and [3] are the only grammatically correct phrases. However, 'according to' is used when something is said or indicated by
someone or something. E.g., 'According to my teacher, we should have exams next week.' We require a phrase that indicates compliance
with the provisions of the Labour Law - a replacement of 'as per'. So, [1] is incorrect in context of the above sentence. 'In accordance with'
is used to indicate compliance with a certain body or rules and is the correct option. Hence, [3].
Question: 168

IMS Learning Resources Pvt.Ltd.,Mumbai.All copyrights to this material vestswith IMS Learning Resources Pvt.Ltd.
No part of this materials either in part oras a whole shall be copied,printed,electronically reproduced,sold or distributed without the written
consent of IMS Learing Resources Pvt.Ltd.and any such violation would entail initiation of suitable legal proceedings.

Copyright

Login ID:8FA03376/Student Name:ANSHULGARG/Overall Score:7

The sentence below has a highlighted phrase. Replace the phrase with the correct option from the choices given below.
The condition called selfitis is defined as the compulsive desirable obsession to take photos of one's self and post them on social media
as a way to make up for lack of self-esteem and fill a gap in intimacy.
compulsive desirable obsession

2)

desirable obsessive compulsion

3)

obsessively compulsory desire

4)

obsessive compulsive desire

ite
d

1)

Li
m

Explanation:
As the sentence talks about a medical condition, it cannot be 'desirable'. So, [1] and [2] can be ruled out. A desire cannot be 'obsessively
compulsory' or 'compulsively obsessed'; it can be 'obsessive' (persistent) and 'compulsive' (uncontrollable). Hence, [4].

IM

Le

ar

ni

ng

es

ou

rc

es

Pr

iv
at
e

Question: 169
Read the following passage and answer the questions that follow.
One idea in the study of emotion and its impact on psychological health is overdue for retirement: that negative emotions (like sadness or
fear) are inherently bad or maladaptive for our psychological well-being, and positive emotions (like happiness or joy) are inherently good
or adaptive. Such value judgments are to be understood, within the framework of affective science, as depending on whether an emotion
impedes or fosters a person's ability to pursue goals, attain resources, and function effectively within society. Claims of the sort 'sadness
is inherently bad' or 'happiness is inherently good' must be abandoned in light of burgeoning advances in the scientific study of human
emotion.
Let's start with negative emotions. Treatments like cognitive-behavioural therapy focus heavily on the reduction of negative feelings and
moods as part of enhancing well-being. Yet a strong, fairly new, body of scientific work suggests that negative emotions are essential to
our psychological well-being. Here are 3 examples. First, from an evolutionary perspective, negative emotions aid in our survival they
provide important clues to threats or problems that need our attention (such as an unhealthy relationship or a dangerous situation).
Second, negative emotions help us focus: they facilitate more detailed and analytic thinking, reduce stereotypic thinking, enhance
eyewitness memory, and promote persistence on challenging cognitive tasks. Third, attempting to thwart or suppress negative emotions
rather than accept and appreciate them paradoxically backfires and increases feelings of distress and intensifies clinical symptoms of
substance abuse, overeating, and even suicidal ideation. Negative emotions are hence not inherently bad for us. Moreover, the relative
absence of them predicts poorer psychological adjustment.
Positive emotions have been conceptualized as pleasant or positively valenced states that motivate us to pursue goal-directed behaviour.
The popular as well as scientific assumption has always been that positive emotional states should always be maximized. But recently,
there has been a mounting body of work against the claim that positive emotions are inherently good. First, positive emotions foster more
self-focused behaviour, including increased selfishness, greater stereotyping of out-group members, increased cheating and dishonesty,
and decreased empathic accuracy in some contexts. Second, positive emotions are associated with greater distractibility and impaired
performance on detail-oriented cognitive tasks. Third, because positive emotion may promote decreased inhibition it has been associated
with greater risk-taking behaviours and mortality rates. Indeed, the presence of positive emotions is not always adaptive and sometimes
can impede our well-being and even survival.
We are left to conclude that valence is not value: we cannot infer value judgments about emotions on the basis of their positive or negative
valence. There is no intrinsic goodness or badness of an emotion merely because of its positivity or negativity, respectively. Instead, we
must refine specific value-based determinants for an emotion's functionality. Also, the degree of psychological flexibility someone
possesses including how quickly one can shift emotions or rebound from a stressful situation promotes critical clinical health outcomes.
Likewise, we find that psychological well-being is not entirely determined by the presence of one type or kind of an emotion but rather an
ability to experience a rich diversity of both positive and negative emotions. Whether or not an emotion is 'good' or 'bad' seems to have
surprisingly little to do with the emotion itself, but rather how mindfully we ride the ebbs and tides of our rich emotional life.
What is the author's attitude towards negative emotions like sadness in this passage?
1)

Questioning

2)

Speculative

IMS Learning Resources Pvt.Ltd.,Mumbai.All copyrights to this material vestswith IMS Learning Resources Pvt.Ltd.
No part of this materials either in part oras a whole shall be copied,printed,electronically reproduced,sold or distributed without the written
consent of IMS Learing Resources Pvt.Ltd.and any such violation would entail initiation of suitable legal proceedings.

Copyright

Login ID:8FA03376/Student Name:ANSHULGARG/Overall Score:7

3)

Dismissive

4)

Accepting

Explanation:
According to the author, negative emotions like sadness, fear, etc. are not necessarily bad in themselves - they can serve useful
purposes. She does not question their value, nor does she speculate about them - rather, she states scientific facts on the basis of which
she accepts the value of negative emotions. [3] is the opposite of the author's attitude. Hence, [4].

ite
d

Question: 170
Read the following passage and answer the questions that follow.

The Value of Emotions

2)

A New Perspective on Emotions

3)

Negative Emotions vs. Positive Emotions

4)

The Scientific Basis for Emotions

iv
at
e

1)

Li
m

Choose a suitable title for this passage.

2)

Only [b]

3)

Both [a] and [b]

4)

Both [a] and [c]

ni

Only [a]

ar

1)

ng

es

ou

Question: 171
Read the following passage and answer the questions that follow.

rc

es

Pr

Explanation:
The passage is not about the value of emotions in general or a theory regarding them, so [1] can be ruled out at once. Though the author
does take a scientific look at emotions, that is not the main topic of the passage. The 'vs.' in [3] suggests an opposition between negative
emotions and positive emotions that is not present in the passage - the author claims that both are necessary for people's psychological
well-being. [2] is the most suitable title, as the author presents a new perspective on emotions by showing that positive emotions are not
necessarily good for one, and negative emotions are not necessarily bad. (Refer to the last sentence of the first paragraph and the second
sentence of the second paragraph). Hence, [2].

IM

Le

Explanation:
The author's statement 'valence is not value' in the last paragraph sums up the main point of her argument. She argues that 'there is no
intrinsic goodness or badness of an emotion merely because of its positivity or negativity, respectively' i.e. whether an emotion is
positive or negative does not determine its usefulness. So [a] is correct. But [b] is an exaggeration the author does not mean that all
positive emotions are bad and all negative emotions are good. [c] is irrelevant, as the passage does not mention the issue about the
morality of people at all (i.e. 'good or bad people'). Hence, [1].
Question: 172
Read the following passage and answer the questions that follow.
Assume that an experiment showed that friendly people are more likely to be better adjusted than unfriendly ones. How would this impact
the author's argument?
1)

It would probably strengthen author's argument.

IMS Learning Resources Pvt.Ltd.,Mumbai.All copyrights to this material vestswith IMS Learning Resources Pvt.Ltd.
No part of this materials either in part oras a whole shall be copied,printed,electronically reproduced,sold or distributed without the written
consent of IMS Learing Resources Pvt.Ltd.and any such violation would entail initiation of suitable legal proceedings.

Copyright

Login ID:8FA03376/Student Name:ANSHULGARG/Overall Score:7

2)

It would probably weaken the author's argument.

3)

It would definitely weaken the author's argument.

4)

It is not possible to tell if it would affect the author's argument.

Explanation:
The author's argument is about positive and negative emotions, whereas 'friendly' and 'unfriendly' are personality types, not emotions.
Since there is no information in the passage regarding personality types, we cannot tell whether the information in the question would
affect the author's argument. Hence, [4].

Which of the following cannot be inferred from the passage?

Li
m

ite
d

Question: 173
Read the following passage and answer the questions that follow.

Happy people are more likely to give money to charity than sad ones.

2)

A frightened person makes a better eyewitness to a crime than a cheerful one.

3)

Happy people are more likely to participate in dangerous sports than depressed ones.

4)

A person who has never experienced sadness or fear is likely to be psychologically maladjusted.

iv
at
e

1)

rc

es

Pr

Explanation:
According to the passage, negative emotions 'enhance eyewitness memory', so [2] can be inferred. The passage also states that positive
emotion is 'associated with greater risk-taking behaviours', so [3] also seems likely. [4] can be inferred from the fact that 'the relative
absence of [negative emotions] predicts poorer psychological adjustment'. Only [1] is not inferable from the passage: according to the
passage, 'positive emotions foster more self-focused behaviour, including increased selfishness', so happy people are less likely to give
money to charity than sad ones. Hence, [1].

ou

Question: 174
The sentence below has a highlighted phrase. Replace the phrase with the correct option from the choices given below.

es

Israel faced international criticism after announcing that it would seize control of almost 1,000 acres of Palestinian-claimed land for the
expected construction of what would be its largest new settlement in 30 years.
seize control of

2)

cease control of

3)

control the seizure of

4)

seize the control of

ni

ng

1)

Le

ar

Explanation:
The sentence talks about taking control of land, therefore the correct word is 'seize', not 'cease' which means 'stop'. This rules out option
[2]. The sentence implies that Israel was (itself) planning to seize land, not control the action of seizing. Hence, option [3] is incorrect.
'Seize control of' is the correct phrase; 'the' is not put before 'control' as the latter is not used in a specific sense. Hence, [1].

IM

Question: 175
The sentence below has a highlighted phrase. Replace the phrase with the correct option from the choices given below.
It was largely engineers such as John Smeaton, Peter Ewart, Carl Holtzmann, Gustave-Adolphe Hirn and Marc Seguin who object to
conservation of momentum alone was not adequate for practical calculation and made use of Leibniz's principle.
1)

object that

2)

objected that

3)

object to

IMS Learning Resources Pvt.Ltd.,Mumbai.All copyrights to this material vestswith IMS Learning Resources Pvt.Ltd.
No part of this materials either in part oras a whole shall be copied,printed,electronically reproduced,sold or distributed without the written
consent of IMS Learing Resources Pvt.Ltd.and any such violation would entail initiation of suitable legal proceedings.

Copyright

Login ID:8FA03376/Student Name:ANSHULGARG/Overall Score:7

4)

objected to

Explanation:
The sentence is refers to the subject (the engineers) in the past, therefore the corresponding verb should be in the same tense. Options
[1] and [3] can be ruled out since 'object' is used in the present tense. Option [4] uses the incorrect preposition ('to'). 'Objected to' should
be followed by a noun/noun phrase but the underlined phrase is followed by a clause. Hence, [2].

ite
d

Question: 176
The sentence below has a highlighted phrase. Replace the phrase with the correct option from the choices given below.

before he segued

2)

before they segue

3)

before segueing

4)

before it segues

iv
at
e

1)

Li
m

The organist's fingers step lightly through a world of summer fruits, picking out high notes that conjure zest and vitality, before it segues
into a lazy melody of golden malt fields.

Pr

Explanation:
The subject of the verb 'segue' (i.e. to make a transition from one thing to another smoothly and without interruption) should be the
organist's fingers. To maintain parallelism, the verbs 'picking' and 'segueing' should be in the same form. Hence, [3].

es

Question: 177
In each of the following sentences there are two blank spaces. Below each sentence there are four options. Find out which options can be
filled in the blanks in the sentence in the same sequence to make the sentence grammatically correct and meaningful.

souled, reluctance

2)

blooded, temperance

3)

minded, suspicion

4)

armed, reverence

es

1)

ou

rc

Though he was a hot ________ pugilist, he always urged __________ among his friends.

ni

ng

Explanation:
The sentence begins with a 'though'. From this we can say that the two words in the blank have to be contrasting in nature. The general
collocation is 'hot blooded'. In the second blank the word that comes should suggest control of anger. Thus, the most appropriate word in
this blank is 'temperance'. Hence, the correct pair of word is 'blooded' and 'temperance'. Hence, [2].

Le

ar

Question: 178
In each of the following sentences there are two blank spaces. Below each sentence there are four options. Find out which options can be
filled in the blanks in the sentence in the same sequence to make the sentence grammatically correct and meaningful.

grappling, help

IM

1)

For Indian consumers __________ under high food prices, it might be small __________ that the rest of the world too is sharing the
problem in varying degrees.

2)

dying, support

3)

reeling, comfort

4)

buried, dilemma

Explanation:
One 'reels' under something or one is 'buried' under something. Thus, the first blank can take 'reeling' or 'buried'. From the context of the
passage we can see that it means to suggest that the consumers can breathe a sigh of relief. Thus, the word 'comfort' is better than

IMS Learning Resources Pvt.Ltd.,Mumbai.All copyrights to this material vestswith IMS Learning Resources Pvt.Ltd.
No part of this materials either in part oras a whole shall be copied,printed,electronically reproduced,sold or distributed without the written
consent of IMS Learing Resources Pvt.Ltd.and any such violation would entail initiation of suitable legal proceedings.

Copyright

Login ID:8FA03376/Student Name:ANSHULGARG/Overall Score:7

'dilemma' for the second blank. With 'grappling', the correct preposition is 'with'. Hence, it is incorrect. Hence, [3].
Question: 179
Read the following passage and answer the questions that follow.

rc

es

Pr

iv
at
e

Li
m

ite
d

The intention behind the government's decision to ban employment of children below 14 years of age as domestic servants, and as
helpers in the unorganised part of the hospitality industry, under the Child Labour (Prohibition and Regulation) Act, 1986, is doubtless
noble.
But can that translate on the ground? Laws can be enforced only when they accord with real social processes. Sadly, large swathes of the
country are in the grip of primitive socio-economic formations where child labour is a normal phenomenon.
Childhood in such a social order is a period of apprenticeship in workshops and fields. Work and education in such societies are not
clearly differentiated. A law that seeks to ban child labour would, as far as such communities go, have little chance of success.
Laws would become enforceable only when political parties intervene at the grassroots to transform traditional socio-economic formations.
Separation of education and childhood development on the one hand, and work and adulthood on the other is a feature of the modern
industrial economy. That is desirable, not only because it suits modern production, but also because it spells a higher level of social
development.
The government, together with the political class, should do much more than simply ban child labour. Its failure to monitor, and rehabilitate
children working in sectors where the ban is already in force does not inspire much confidence.
The inadequate government schooling system gives poverty-struck members of traditional social formations every reason to continue to
send their children to work instead of putting them in schools.
The Indian political-legislative process would become truly acceptable to all only when it manages to effect a convergence between local
needs, and universally-desired and legally-ordained rights.
The recent ban does make the child's right not to work justiciable. That could facilitate a new politics for social development. There's just
one problem: such politics continues to be invisible.
What is the primary purpose of the author?

To explain how child labour laws can be made more effective

2)

To state the main reasons for child labour

3)

To discuss steps to eradicate child labour

4)

To find flaws with the existing government laws related to child labour

es

ou

1)

ar

ni

ng

Explanation:
At various points in the passage, it is clearly stated how child labour related laws can be made more effective. Even though there is
mention of some reasons behind child labour and some flaws in the government rules, these don't form the primary purpose of the author.
The author has not discussed proper steps to eradicate child labour. He has just given some general guidelines to make laws that do that
more effectively. Hence, [4].

Le

Question: 180
Read the following passage and answer the questions that follow.

The socio-economic conditions of the people

IM

1)

Which of the following has not been cited as one of the reasons for continued child labour?

2)

The schooling facilities available

3)

The government's inability to enforce the law at the grassroots level

4)

Children's willingness to earn money at an early age

Explanation:
All the options except [4] are discussed in the passage. Hence, [4].

IMS Learning Resources Pvt.Ltd.,Mumbai.All copyrights to this material vestswith IMS Learning Resources Pvt.Ltd.
No part of this materials either in part oras a whole shall be copied,printed,electronically reproduced,sold or distributed without the written
consent of IMS Learing Resources Pvt.Ltd.and any such violation would entail initiation of suitable legal proceedings.

Copyright

Login ID:8FA03376/Student Name:ANSHULGARG/Overall Score:7

Question: 181
Read the following passage and answer the questions that follow.
Which is the most likely reason for banning child labour particularly in the unorganized part of the hospitality industry?
Child labour is high in this sector because jobs are easily available.

2)

This sector doesn't have any education-related criteria for hiring.

3)

This sector requires unskilled labour for menial jobs, therefore hiring children is more feasible.

4)

Cannot be determined

ite
d

1)

Li
m

Explanation:
The passage does not discuss the reasons behind the ban for the hospitality industry. Hence, [4].
Question: 182
Read the following passage and answer the questions that follow.

iv
at
e

The passage mentions 'translate on the ground'. What measures should the government take to make this possible?
Make laws such that they are coherent with the current social processes

2)

Have strict mechanisms to monitor the sectors where the ban is already enforced

3)

Provide a better education system for children of the lower strata

4)

All of the above

Pr

1)

rc

es

Explanation:
'Translate on the ground', in the context of the passage, refers to the successful implementation of the law. All the reasons mentioned in
options [1], [2] and [3] are given in the passage. Hence, [4].

IM

Le

ar

ni

ng

es

ou

Question: 183
Read the following passage and answer the questions that follow.
The intention behind the government's decision to ban employment of children below 14 years of age as domestic servants, and as
helpers in the unorganised part of the hospitality industry, under the Child Labour (Prohibition and Regulation) Act, 1986, is doubtless
noble.
But can that translate on the ground? Laws can be enforced only when they accord with real social processes. Sadly, large swathes of the
country are in the grip of primitive socio-economic formations where child labour is a normal phenomenon.
Childhood in such a social order is a period of apprenticeship in workshops and fields. Work and education in such societies are not
clearly differentiated. A law that seeks to ban child labour would, as far as such communities go, have little chance of success.
Laws would become enforceable only when political parties intervene at the grassroots to transform traditional socio-economic formations.
Separation of education and childhood development on the one hand, and work and adulthood on the other is a feature of the modern
industrial economy. That is desirable, not only because it suits modern production, but also because it spells a higher level of social
development.
The government, together with the political class, should do much more than simply ban child labour. Its failure to monitor, and rehabilitate
children working in sectors where the ban is already in force does not inspire much confidence.
The inadequate government schooling system gives poverty-struck members of traditional social formations every reason to continue to
send their children to work instead of putting them in schools.
The Indian political-legislative process would become truly acceptable to all only when it manages to effect a convergence between local
needs, and universally-desired and legally-ordained rights.
The recent ban does make the child's right not to work justiciable. That could facilitate a new politics for social development. There's just
one problem: such politics continues to be invisible.
Which word best describes the government?
1)

Lazy

IMS Learning Resources Pvt.Ltd.,Mumbai.All copyrights to this material vestswith IMS Learning Resources Pvt.Ltd.
No part of this materials either in part oras a whole shall be copied,printed,electronically reproduced,sold or distributed without the written
consent of IMS Learing Resources Pvt.Ltd.and any such violation would entail initiation of suitable legal proceedings.

Copyright

Login ID:8FA03376/Student Name:ANSHULGARG/Overall Score:7

2)

Callous

3)

Negligent

4)

Unsympathetic

Explanation:
The passage states that even though the government is aware of the problem of child labour and has made relevant laws, it has failed to
ensure their implementation. So, it has neglected its duty. Thus, [3] is the most appropriate answer. From the passage, we cannot deduce
why the government has been remiss. So, [1], [2] and [4] have no basis in the passage. Hence, [3].

[2]

3)

[3]

4)

[4]

Pr

2)

es

[1]

rc

1)

iv
at
e

Li
m

ite
d

Question: 184
Read the sentences given below carefully. Parts of the sentences have been underlined as [1], [2] and [3]. If there is an error in the
sentence as a result of any of these parts, mark the relevant part. If there are no errors, then mark [4].

es

ou

Explanation:
There is no error in the given sentence. 'Grade inflation' is a valid expression and refers to a rise in the average grade given to students.
Hence, [4].

2)

[2]

3)

[3]
[4]

IM

4)

Le

[1]

1)

ar

ni

ng

Question: 185
Read the sentences given below carefully. Parts of the sentences have been underlined as [1], [2] and [3]. If there is an error in the
sentence as a result of any of these parts, mark the relevant part. If there are no errors, then mark [4].

Explanation:
In [3], the pronoun 'its' should be replaced by 'their' as it refers to the antecedent 'folks'. Hence, [3].
Question: 186
The following paragraph has some words missing. Choose the options that best fill the blanks.
A small eruption has ____(30)____ north of Iceland's Bardarbunga volcano after nearly two weeks of earthquakes in the region, but no

IMS Learning Resources Pvt.Ltd.,Mumbai.All copyrights to this material vestswith IMS Learning Resources Pvt.Ltd.
No part of this materials either in part oras a whole shall be copied,printed,electronically reproduced,sold or distributed without the written
consent of IMS Learing Resources Pvt.Ltd.and any such violation would entail initiation of suitable legal proceedings.

Copyright

Login ID:8FA03376/Student Name:ANSHULGARG/Overall Score:7

1)

occurred

2)

come

3)

risen

4)

materialized

ite
d

volcanic ash has been detected. The ___(31)____ 1-km fissure eruption in the Holuhraun area prompted the Icelandic Met Office to raise
the warning code for aviation to red, the highest level. The rumblings at Iceland's largest volcano system, which is covered by a several
hundred meters thick glacier, __(32)___ raised worries of an eruption that could spell trouble for air travel. In 2010, an ash cloud from the
Eyjafjallajokull volcano closed much of Europe's airspace for six days. The ___(33)____, just after 0000 GMT, took place in a lava field not
___(34)___ by ice. The risk of an ash cloud is highest in case of a sub-glacial eruption.

predicted

2)

guessed

3)

analysed

4)

estimated

Pr

1)

iv
at
e

Question: 187
The following paragraph has some words missing. Choose the options that best fill the blanks.

Li
m

Explanation:
An eruption cannot 'come' or 'rise' or 'materialize' (become actual or real). The sentence is simply trying to say that a small eruption has
taken place or 'occurred'. Hence, [1].

ou

rc

es

Explanation:
The most appropriate word here is 'estimated', as the authorities would estimate the size of the eruption before issuing a suitable warning.
It would be foolish to raisethe warning code on the basis of predictions or guesswork. 'Analysed' is not a suitable adjective to go with '1km'; analysing the length of the eruption doesn't make sense. Hence, [4].

2)

had

3)

have

4)

and

has

ng

1)

es

Question: 188
The following paragraph has some words missing. Choose the options that best fill the blanks.

ni

Explanation:

Le

ar

The subject in the sentence is 'rumblings' which is plural. So, 'has' is incorrect. Also, the
sentence implies that the authorities are worried now, not that they were worried earlier.
So, 'had' is incorrect. 'And' is unsuitable because the portion of the sentence before the
blank is not a clause. Therefore, the correct option is 'have'. Hence, [3].

IM

Question: 189
The following paragraph has some words missing. Choose the options that best fill the blanks.
1)

accident

2)

incident

3)

eruption

4)

evasion

Explanation:

IMS Learning Resources Pvt.Ltd.,Mumbai.All copyrights to this material vestswith IMS Learning Resources Pvt.Ltd.
No part of this materials either in part oras a whole shall be copied,printed,electronically reproduced,sold or distributed without the written
consent of IMS Learing Resources Pvt.Ltd.and any such violation would entail initiation of suitable legal proceedings.

Copyright

Login ID:8FA03376/Student Name:ANSHULGARG/Overall Score:7

The paragraph has been describing an 'eruption' and continues to do so. Though 'incident'
could be used, 'eruption' describes the event more specifically. Hence, [3].

1)

trashed

2)

covered

3)

hooded

4)

nestled

ite
d

Question: 190
The following paragraph has some words missing. Choose the options that best fill the blanks.

Li
m

Explanation:
'Trashed' (damaged) is irrelevant to the passage. 'Nestled' (settled comfortably) is grammatically incorrect. 'Hooded' can be used when
something hood-like covers something else; it can't be used for a field covered with ice. Hence, [2].

2)

Even though

3)

Until

4)

Nevertheless

rc

Whether or not

ou

1)

es

Pr

iv
at
e

Question: 191
The two statements given in each question can be combined into one by using an appropriate starting word or phrase. Choose the most
appropriate starter from the options.

es

Explanation:
Until is used to indicate a condition, which is not relevant to the sentence. Nevertheless doesnt grammatically fit as a sentence opener
when both the sentences are combined. Whether or not and even if make the information in the first sentence seem questionable,
whereas the sentence is assertive. Even though is the best fit. Hence, [2].

Since

IM

2)

As

1)

Le

ar

ni

ng

Question: 192
The two statements given in each question can be combined into one by using an appropriate starting word or phrase. Choose the most
appropriate starter from the options.

3)

Because

4)

A, B and C

Explanation:
Together, the two sentences show how one action (the government's response) has led to a certain consequence (truckers losing jobs
and wages and benefits). This relationship is best expressed using 'because', 'as' or 'since' (they are synonymous). Hence, [4].
Question: 193

IMS Learning Resources Pvt.Ltd.,Mumbai.All copyrights to this material vestswith IMS Learning Resources Pvt.Ltd.
No part of this materials either in part oras a whole shall be copied,printed,electronically reproduced,sold or distributed without the written
consent of IMS Learing Resources Pvt.Ltd.and any such violation would entail initiation of suitable legal proceedings.

Copyright

Login ID:8FA03376/Student Name:ANSHULGARG/Overall Score:7

When

2)

While

3)

During

4)

Since

Explanation:
The two sentences reflect contrasting actions. So, 'while' is the most appropriate option. Hence, [2].

Li
m

1)

ite
d

The two statements given in each question can be combined into one by using an appropriate starting word or phrase. Choose the most
appropriate starter from the options.

If

3)

Although

4)

Even though

es

2)

rc

When

ou

1)

Pr

iv
at
e

Question: 194
The two statements given in each question can be combined into one by using an appropriate starting word or phrase. Choose the most
appropriate starter from the options.

es

Explanation:
The two sentences don't contrast each other or indicate a condition. So, 'if', 'although', 'even though' and 'until' are inappropriate. The
second sentence simply indicates an action that took place at the same time as the action in the first sentence. Hence, [1].

2)

Unless

3)

Moreover

4)

However

ar

Though

Le

1)

ni

ng

Question: 195
The two statements given in each question can be combined into one by using an appropriate starting word or phrase. Choose the most
appropriate starter from the options.

IM

Explanation:
The two sentences contain contrasting information. 'However' cannot begin the combined sentence, so, 'though' is the correct answer.
Hence, [1].
Question: 196
The following paragraph has some words missing. Choose the options that best fill the blanks.
A cloud-free atmosphere has allowed scientists to pick out signs of water vapour on a distant planet the size of Neptune: the ___(40)___
'exoplanet' ever to ___(41)___ its chemical composition. The planet, designated HAT P-11b, orbits a sun in the constellation Cygnus.

IMS Learning Resources Pvt.Ltd.,Mumbai.All copyrights to this material vestswith IMS Learning Resources Pvt.Ltd.
No part of this materials either in part oras a whole shall be copied,printed,electronically reproduced,sold or distributed without the written
consent of IMS Learing Resources Pvt.Ltd.and any such violation would entail initiation of suitable legal proceedings.

Copyright

Login ID:8FA03376/Student Name:ANSHULGARG/Overall Score:7

smallest

2)

largest

3)

shortest

4)

newest

Li
m

1)

ite
d

Scientists' observations ___(42)___ also greatly assisted by there being no clouds on HAT P-11b, which would otherwise have frustrated
their attempts to probe its gaseous envelope.
Commenting on the findings, Dr Eliza Kempton from Grinnell College, said 'Astronomers have detected water vapour in the atmospheres
of larger planets planets that are closer in size to Jupiter. But you can imagine that eventually we want to be able to ___(43)___
molecules in the atmospheres of even smaller planets.
The presence of water has obvious ___(44)___ for life, although HAT P-11b is too close to its star and therefore too hot to be
habitable.

iv
at
e

Explanation:
Options [3] and [4] can be quickly eliminated: 'shortest' is used to indicate height and 'newest' is incorrect as the first blank is followed by
'ever'. 'Smallest' is the most appropriate answer as, subsequently, there is an emphasis on smaller planets. Hence, [1].

present

2)

highlight

3)

reveal

4)

show

es

1)

Pr

Question: 197
The following paragraph has some words missing. Choose the options that best fill the blanks.

ou

rc

Explanation:
The sentence indicates that the chemical composition of the planet has now become known. Therefore, 'reveal' is the most appropriate
word. Hence, [3].

es

Question: 198
The following paragraph has some words missing. Choose the options that best fill the blanks.
are

2)

were

3)

were being

4)

are being

ni

ng

1)

Le

ar

Explanation:
This sentence could use the present perfect tense or the simple past tense. As 'have also been' is not an option, 'were' is the most
appropriate choice. 'Are' indicates a general truth, which is not the case here. Any continuous tense would be incorrect here as there is no
ongoing activity being referred to. The past perfect tense is also inappropriate here as it ought to indicate an action that was completed
before another in the past. Hence, [2].

IM

Question: 199
The following paragraph has some words missing. Choose the options that best fill the blanks.
1)

identify

2)

inspect

3)

ascertain

4)

design

Explanation:

IMS Learning Resources Pvt.Ltd.,Mumbai.All copyrights to this material vestswith IMS Learning Resources Pvt.Ltd.
No part of this materials either in part oras a whole shall be copied,printed,electronically reproduced,sold or distributed without the written
consent of IMS Learing Resources Pvt.Ltd.and any such violation would entail initiation of suitable legal proceedings.

Copyright

Login ID:8FA03376/Student Name:ANSHULGARG/Overall Score:7

The paragraph is about 'findings' and the previous sentence has talked about the 'detection' of water vapour. So, 'identify' is the most
appropriate word for this blank. [3] and [4] make no sense and molecules can be 'inspected' only after they are identified. Note: scientists
could ascertain the presence of molecules, not the molecule themselves. Hence, [1].

1)

meaning

2)

uses

3)

advantages

4)

implications

ite
d

Question: 200
The following paragraph has some words missing. Choose the options that best fill the blanks.

IM

Le

ar

ni

ng

es

ou

rc

es

Pr

iv
at
e

Li
m

Explanation:
The sentence is trying to say that the presence of water, by itself, implies that life is possible, though some other conditions work against
this possibility. Considering the structure of the sentence, only [4] fits. Option [1] implies a philosophical context; [2] and [3] are
grammatically incorrect and presuppose the existence of life. Hence, [4].

IMS Learning Resources Pvt.Ltd.,Mumbai.All copyrights to this material vestswith IMS Learning Resources Pvt.Ltd.
No part of this materials either in part oras a whole shall be copied,printed,electronically reproduced,sold or distributed without the written
consent of IMS Learing Resources Pvt.Ltd.and any such violation would entail initiation of suitable legal proceedings.

Copyright

You might also like